Está en la página 1de 100

Traducido del inglés al español - www.onlinedoctranslator.

com
INTRODUCCIÓN A LA MECÁNICA CUÁNTICA

Tercera edicion

Los cambios y adiciones a la nueva edición de este libro de texto clásico incluyen:

Un nuevo capítulo sobre simetrías y leyes de conservación

Nuevos problemas y ejemplos

Explicaciones mejoradas

Más problemas numéricos para trabajar en una computadora

Nuevas aplicaciones a la física del estado sólido

Tratamiento consolidado de potenciales dependientes del tiempo

David J. Griffiths recibió su BA (1964) y PhD (1970) de la Universidad de Harvard. Enseñó en Hampshire College,
Mount Holyoke College y Trinity College antes de unirse a la facultad de Reed College en 1978. En 2001-2002 fue
profesor visitante de física en las cinco universidades (UMass, Amherst, Mount Holyoke, Smith y Hampshire) , y en
la primavera de 2007 enseñó Electrodinámica en Stanford. Aunque su doctorado fue en teoría de partículas
elementales, la mayor parte de su investigación se centra en electrodinámica y mecánica cuántica. Es autor de
más de cincuenta artículos y cuatro libros:Introducción a la electrodinámica (4a edición, Cambridge University
Press, 2013), Introducción a las partículas elementales (2a edición, Wiley-VCH, 2008), Introducción a la mecánica
cuántica (2a edición, Cambridge, 2005), y Revoluciones en la física del siglo XX(Cambridge, 2013).

Darrell F. Schroeter es un teórico de la materia condensada. Recibió su licenciatura (1995) de Reed College y su

doctorado (2002) de la Universidad de Stanford, donde fue becario de investigación de posgrado de la National Science

Foundation. Antes de unirse a la facultad de Reed College en 2007, Schroeter enseñó tanto en Swarthmore College

como en Occidental College. Su historial de investigación teórica exitosa con estudiantes de pregrado fue reconocido en

2011 cuando fue nombrado becario de KITP-Anacapa.

2
INTRODUCCIÓN A QUANTUM
MECÁNICA
Tercera edicion

DAVID J. GRIFFITHS y DARRELL F. SCHROETER


Reed College, Oregón

3
University Printing House, Cambridge CB2 8BS, Reino Unido

One Liberty Plaza, 20th Floor, Nueva York, NY 10006, EE. UU.

477 Williamstown Road, Port Melbourne, VIC 3207, Australia

314–321, 3rd Floor, Plot 3, Splendor Forum, Jasola District Centre, Nueva Delhi - 110025, India

79 Anson Road, # 06–04 / 06, Singapur 079906

Cambridge University Press forma parte de la Universidad de Cambridge.

Fomenta la misión de la Universidad al difundir el conocimiento en la búsqueda de la educación, el aprendizaje y la investigación al más alto nivel.

niveles internacionales de excelencia.

www.cambridge.org

Información sobre este título: www.cambridge.org/9781107189638

DOI: 10.1017 / 9781316995433

Segunda edición © David Griffiths 2017

Tercera edición © Cambridge University Press 2018

Esta publicación está protegida por derechos de autor. Sujeto a la excepción legal y a las disposiciones de los convenios colectivos de licencia pertinentes, no

la reproducción de cualquier parte puede tener lugar sin el permiso por escrito de Cambridge University Press.

Este libro fue publicado anteriormente por Pearson Education, Inc. 2004

Segunda edición reeditada por Cambridge University Press 2017

Tercera edición 2018

Impreso en el Reino Unido por TJ International Ltd. Padstow Cornwall, 2018

Un registro del catálogo de esta publicación está disponible en la Biblioteca Británica.

Datos de catalogación en publicación de la Biblioteca del

CongresoNombres: Griffiths, David J. | Schroeter, Darrell F.

Título: Introducción a la mecánica cuántica / David J. Griffiths (Reed College, Oregon), Darrell F. Schroeter (Reed College, Oregon).

Descripción: Tercera edición. | bla: Cambridge University Press, 2018.

Identificadores: LCCN 2018009864 | ISBN 9781107189638

Materias: LCSH: Teoría cuántica.

Clasificación: LCC QC174.12 .G75 2018 | DDC 530.12 – dc23

Registro LC disponible en https://lccn.loc.gov/2018009864

ISBN 978-1-107-18963-8 Tapa dura

Recursos adicionales para esta publicación en www.cambridge.org/IQM3ed

Cambridge University Press no se hace responsable de la persistencia o precisión de las URL de sitios web de Internet externos o de terceros.

mencionado en esta publicación y no garantiza que el contenido de dichos sitios web sea, o seguirá siendo, exacto o apropiado.

4
5
Contenido

Prefacio

I Teoría

1 La función de onda
1.1 La ecuación de Schrödinger
1.2 La interpretación estadística
1.3 Probabilidad

1.3.1 Variables discretas


1.3.2 Variables continuas
1.4 Normalización
1,5 Impulso
1,6 El principio de incertidumbre
Otros problemas en el capítulo 1

2 Ecuación de Schrödinger independiente del tiempo

2.1 Estados estacionarios

2.2 El pozo cuadrado infinito


2.3 El oscilador armónico
2.3.1 Método algebraico
2.3.2 Método analítico
2.4 La partícula libre
2.5 El potencial de la función delta
2.5.1 Estados limitados y estados de dispersión

2.5.2 El pozo de función delta


2.6 El cuadrado finito más
problemas en el capítulo 2

3 Formalismo

3.1 Espacio Hilbert

3.2 Observables
3.2.1 Operadores hermitianos

3.2.2 Estados determinados

3.3 Funciones propias de un operador hermitiano

3.3.1 Espectros discretos

3.3.2 Espectros continuos

6
3.4 Interpretación estadística generalizada
3,5 El principio de incertidumbre

3.5.1 Prueba del principio de incertidumbre generalizada

3.5.2 El paquete de ondas de mínima incertidumbre

3.5.3 El principio de incertidumbre energía-tiempo

3.6 Vectores y operadores


3.6.1 Bases en Hilbert Space
3.6.2 Notación de Dirac

3.6.3 Cambio de bases en la notación de Dirac

Problemas adicionales en el capítulo 3

4 Mecánica cuántica en tres dimensiones


4.1 La ecuación de Schröger
4.1.1 Coordenadas esféricas
4.1.2 La ecuación angular
4.1.3 La ecuación radial
4.2 El átomo de hidrógeno
4.2.1 La función de onda radial
4.2.2 El espectro del hidrógeno
4.3 Momento angular
4.3.1 Autovalores
4.3.2 Funciones propias

4.4 Girar
4.4.1 Girar 1/2
4.4.2 Electrón en un campo magnético

4.4.3 Adición de momento angular


4.5 Interacciones electromagnéticas

4.5.1 Acoplamiento mínimo

4.5.2 El efecto Aharonov-Bohm


Más problemas en el capítulo 4

5 Partículas idénticas
5.1 Sistemas de dos partículas

5.1.1 Bosones y fermiones


5.1.2 Fuerzas de cambio

5.1.3 Girar
5.1.4 Principio de simetrización generalizada
5.2 Átomos

5.2.1 Helio
5.2.2 La tabla periodica
5.3 Sólidos

5.3.1 El gas de electrones libres

5.3.2 Estructura de la banda

Más problemas en el capítulo 5

7
6 Simetrías y leyes de conservación
6.1 Introducción
6.1.1 Transformaciones en el espacio

6.2 El operador de traducción


6.2.1 Cómo se transforman los operadores

6.2.2 Simetría traslacional


6.3 Leyes de conservación

6.4 Paridad

6.4.1 Paridad en una dimensión


6.4.2 Paridad en tres dimensiones
6.4.3 Reglas de selección de paridad

6.5 Simetría rotacional


6.5.1 Rotaciones sobre el eje z
6.5.2 Rotaciones en tres dimensiones
6.6 Degeneración

6,7 Reglas de selección rotacional

6.7.1 Reglas de selección para operadores escalares

6.7.2 Reglas de selección para operadores vectoriales

6,8 Traducciones en el tiempo

6.8.1 La imagen de Heisenberg

6.8.2 Invarianza de la traducción del tiempo

Otros problemas en el capítulo 6

II Aplicaciones

7 Teoría de la perturbación independiente del tiempo

7.1 Teoría de la perturbación no generada


7.1.1 Formulación general
7.1.2 Teoría de primer orden

7.1.3 Energías de segundo orden

7.2 Teoría de la perturbación degenerada

7.2.1 Degeneración doble


7.2.2 "Estados buenos

7.2.3 Degeneración de orden superior

7.3 La fina estructura del hidrógeno


7.3.1 La corrección relativista
7.3.2 Acoplamiento de giro-órbita

7.4 El efecto Zeeman


7.4.1 Efecto Zeeman de campo débil

7.4.2 Efecto Zeeman de campo fuerte

7.4.3 Efecto Zeeman de campo intermedio


7.5 División hiperfina en hidrógeno

8
Más problemas en el capítulo 7

8 El principio variante
8.1 Teoría
8.2 El estado fundamental del helio
8.3 El ion de la molécula de hidrógeno

8.4 La molécula de hidrógeno, más


problemas en el capítulo 8

9 La aproximación WKB
9.1 La región "clásica"
9.2 Tunelización

9.3 Las fórmulas de conexión plantean

más problemas en el capítulo 9

10 Dispersión
10.1 Introducción
10.1.1 Teoría clásica de la dispersión
10.1.2 Teoría de la dispersión cuántica

10,2 Análisis de onda parcial


10.2.1 Formalismo
10.2.2 Estrategia

10,3 Cambios de fase

10,4 La aproximación nacida


10.4.1 Forma integral de la ecuación de Schrödinger
10.4.2 La aproximación del primogénito
10.4.3 The Born Series Más
problemas en el capítulo 10

11 Dinámica cuántica
11,1 Sistemas de dos niveles

11.1.1 El sistema perturbado


11.1.2 Teoría de la perturbación dependiente del tiempo

11.1.3 Perturbaciones sinusoidales

11,2 Emisión y absorción de radiación


11.2.1 Ondas electromagnéticas
11.2.2 Absorción, emisión estimulada y emisión espontánea
11.2.3 Perturbaciones incoherentes

11,3 Emisión espontánea


11.3.1 De Einstein A y B Coeficientes
11.3.2 La vida de un estado excitado
11.3.3 Reglas de selección

11,4 Regla de oro de Fermi


11,5 La aproximación adiabática
11.5.1 Procesos adiabáticos

9
11.5.2 El teorema adiabático
Otros problemas en el capítulo 11

12 Epílogo
12,1 La paradoja de EPR
12,2 Teorema de Bell
12,3 Estados mixtos y matriz de densidad
12.3.1 Estados puros

12.3.2 Estados mixtos

12.3.3 Subsistemas
12,4 El teorema de la no clonación

12,5 El gato de Schrödinger

Apéndice Álgebra lineal


A.1 Vectores
A.2 Productos internos

A.3 Matrices
A.4 Cambio de bases

A.5 Autovectores y autovalores


A.6 Transformaciones hermitianas

Índice

10
Prefacio

A diferencia de la mecánica de Newton, la electrodinámica de Maxwell o la relatividad de Einstein, la teoría cuántica no fue

creada, ni siquiera definitivamente empaquetada, por un solo individuo, y conserva hasta el día de hoy algunas de las cicatrices

de su estimulante pero traumática juventud. No existe un consenso general sobre cuáles son sus principios fundamentales,

cómo deben enseñarse o qué es lo que realmente "significa". Todo físico competente puede "hacer" mecánica cuántica, pero las

historias que nos contamos sobre lo que estamos haciendo son tan diversas como las de Scheherazade, y casi tan inverosímiles.

Niels Bohr dijo: "Si no está confundido por la física cuántica, entonces no la ha entendido realmente"; Richard Feynman

comentó: "Creo que puedo decir con seguridad que nadie comprende la mecánica cuántica".

El propósito de este libro es enseñarle cómo hacer mecánica cuántica. Aparte de algunos antecedentes esenciales

en el capítulo1, las cuestiones cuasi filosóficas más profundas se reservan para el final. No creemos que se pueda

discutir inteligentemente lo que la mecánica cuánticamedio hasta que uno tenga un sentido firme de lo que la mecánica

cuántica lo hace. Pero si no puede esperar en absoluto, lea el epílogo inmediatamente después de terminar el capítulo.1

La teoría cuántica no solo es conceptualmente rica, sino que también es técnicamente difícil, y las soluciones exactas para

todos los ejemplos de libros de texto, excepto los más artificiales, son pocas y distantes entre sí. Por tanto, es fundamental

desarrollar técnicas especiales para abordar problemas más realistas. En consecuencia, este libro se divide en dos partes;1 Parte

Icubre la teoría básica, y la Parte II reúne un arsenal de esquemas de aproximación, con aplicaciones ilustrativas. Aunque es

importante mantener las dos parteslógicamente por separado, no es necesario estudiar el material en el orden que se presenta

aquí. Algunos instructores, por ejemplo, pueden desear tratar la teoría de la perturbación independiente del tiempo

inmediatamente después del capítulo.2.

Este libro está diseñado para un curso de un semestre o un año en el nivel junior o senior. Un curso de un

semestre tendrá que concentrarse principalmente en la ParteI; un curso de un año completo debe tener espacio para

material complementario más allá de la ParteII. El lector debe estar familiarizado con los rudimentos del álgebra lineal

(como se resume en el Apéndice), los números complejos y el cálculo ascendente a través de derivadas parciales; algún

conocimiento del análisis de Fourier y la función delta de Dirac ayudaría. La mecánica clásica elemental es esencial, por

supuesto, y un poco de electrodinámica sería útil en algunos lugares. Como siempre, cuanta más física y matemáticas

sepa, más fácil será y más sacará de su estudio. Pero la mecánica cuántica no es algo que fluya suave y naturalmente de

teorías anteriores. Por el contrario, representa un alejamiento abrupto y revolucionario de las ideas clásicas,

provocando una forma totalmente nueva y radicalmente contraria a la intuición de pensar sobre el mundo. Eso, de

hecho, es lo que lo convierte en un tema tan fascinante.

A primera vista, este libro puede parecerle tremendamente matemático. Encontramos polinomios de Legendre,

Hermite y Laguerre, armónicos esféricos, funciones de Bessel, Neumann y Hankel, funciones de Airy e incluso la función

zeta de Riemann, sin mencionar las transformadas de Fourier, los espacios de Hilbert, los operadores hermitianos y los

coeficientes de Clebsch-Gordan. ¿Es todo este equipaje realmente necesario? Quizás no, pero la física es como la

carpintería: usar la herramienta adecuada hace que el trabajomás fácil, no más difícil, y enseñar mecánica cuántica sin el

equipo matemático adecuado es como que le extraigan un diente con un par de pinzas: es posible, pero doloroso. (Por

otro lado, puede resultar tedioso y divertido si el instructor se siente obligado a dar lecciones detalladas sobre el uso

adecuado de cada herramienta. Nuestro instinto es darles palas a los alumnos y decirles que

11
empezar a cavar. Pueden desarrollar ampollas al principio, pero todavía creemos que esta es la forma más eficiente y

emocionante de aprender). En cualquier caso, podemos asegurarle que no hayprofundo matemáticas en este libro, y si

te encuentras con algo desconocido y no encuentras nuestra explicación adecuada, por supuesto pedir alguien al

respecto, o búsquelo. Hay muchos libros buenos sobre métodos matemáticos; recomendamos especialmente Mary

Boas,Métodos matemáticos en las ciencias físicas, Tercera edición, Wiley, Nueva York (2006), o George Arfken y Hans-

Jurgen Weber, Métodos matemáticos para físicos, 7ª ed., Academic Press, Orlando (2013). Pero hagas lo que hagas, no

dejes que las matemáticas, que, para nosotros, son sólo unherramienta—Obscure la física.

Varios lectores han notado que hay menos ejemplos resueltos en este libro de lo que es habitual, y que parte del material

importante está relegado a los problemas. Esto no es un accidente. No creemos que pueda aprender mecánica cuántica sin

hacer muchos ejercicios por sí mismo. Los instructores deben, por supuesto, repasar tantos problemas en clase como el tiempo

lo permita, pero se debe advertir a los estudiantes que este no es un tema sobre el cualalgunauno tiene intuiciones naturales:

aquí está desarrollando un conjunto de músculos completamente nuevo, y simplemente no hay sustituto para la calistenia. Mark

Semon sugirió que ofrezcamos una "Guía Michelin" para los problemas, con diferentes números de estrellas para indicar el nivel

de dificultad e importancia. Parecía una buena idea (aunque, al igual que la calidad de un restaurante, la importancia de un

problema es en parte una cuestión de gustos); hemos adoptado el siguiente esquema de calificación:

un esencial problema que todo lector debería estudiar;

un problema algo más difícil o periférico;

un problema inusualmente desafiante, que puede llevar más de una hora.

(Sin estrellas significa comida rápida: está bien si tienes hambre, pero no muy nutritivo). La mayoría de los problemas de una estrella

aparecen al final de la sección correspondiente; la mayoría de los problemas de tres estrellas se encuentran al final del capítulo. Si se

requiere una computadora, colocamos un mouse en el margen. Un manual de solución está disponible (solo para instructores) del editor.

En la preparación de esta tercera edición hemos intentado conservar en la medida de lo posible el espíritu de la

primera y la segunda. Aunque ahora hay dos autores, todavía usamos el singular ("yo") para dirigirnos al lector; se

siente más íntimo y, después de todo, solo uno de nosotros puede hablar a la vez ("nosotros" en el texto significa usted,

el lector, y yo, el autor, trabajando juntos). Schroeter aporta la nueva perspectiva de un teórico del estado sólido, y es en

gran parte responsable del nuevo capítulo sobre simetrías. Hemos agregado una serie de problemas, aclarado muchas

explicaciones y revisado el epílogo. Pero estábamos decididos a no permitir que el libro engordara, y por esa razón

hemos eliminado el capítulo sobre la aproximación adiabática (se han incorporado puntos importantes de ese capítulo

en el capítulo11), y material eliminado del Capítulo 5 sobre mecánica estadística (que pertenece propiamente a un libro

sobre física térmica). No hace falta decir que los instructores pueden cubrir otros temas que consideren oportunos, pero

queremos que el libro de texto en sí represente el núcleo esencial de la asignatura.

Nos hemos beneficiado de los comentarios y consejos de muchos colegas, que leyeron el manuscrito
original, señalaron debilidades (o errores) en las dos primeras ediciones, sugirieron mejoras en la presentación y
proporcionaron problemas interesantes. Agradecemos especialmente a PK Aravind (Worcester Polytech), Greg
Benesh (Baylor), James Bernhard (Puget Sound), Burt Brody (Bard), Ash Carter (Drew), Edward Chang
(Massachusetts), Peter Collings (Swarthmore), Richard Crandall ( Reed), Jeff Dunham (Middlebury), Greg Elliott
(Puget Sound), John Essick (Reed), Gregg Franklin (Carnegie Mellon), Joel Franklin (Reed),

12
Henry Greenside (Duke), Paul Haines (Dartmouth), JR Huddle (Marina), Larry Hunter (Amherst), David
Kaplan (Washington), Don Koks (Adelaide), Peter Leung (Portland State), Tony Liss (Illinois), Jeffry Mallow
(Chicago Loyola), James McTavish (Liverpool), James Nearing (Miami), Dick Palas, Johnny Powell (Reed),
Krishna Rajagopal (MIT), Brian Raue (Florida Internacional), Robert Reynolds (Reed), Keith Riles (Michigan) ),
Klaus Schmidt-Rohr (Brandeis), Kenny Scott (Londres), Dan Schroeder (Weber State), Mark Semon (Bates),
Herschel Snodgrass (Lewis y Clark), John Taylor (Colorado), Stavros Theodorakis (Chipre), AS Tremsin
(Berkeley), Dan Velleman (Amherst), Nicholas Wheeler (Reed), Scott Willenbrock (Illinois), William Wootters
(Williams) y Jens Zorn (Michigan).

1 Esta estructura se inspiró en el texto clásico de David Park. Introducción a la teoría cuántica, 3ª ed., McGraw-Hill, Nueva York (1992).

13
Parte I
Teoría

14
1
La función de onda

15
1.1 La ecuación de Schrödinger

Imagina una partícula de masa metro, limitado a moverse a lo largo del X eje, sujeto a una fuerza especificada (Figura 1.1). El

programa declásico La mecánica consiste en determinar la posición de la partícula en un momento dado:

. Una vez que sabemos eso, podemos calcular la velocidad , el momento , los
energía cinética , o cualquier otra variable dinámica de interés. Y como vamos
determinando ? Aplicamos la segunda ley de Newton:. (Paraconservador sistemas, el único tipo que
considerará, y, afortunadamente, el único tipo que ocurrir a nivel microscópico: la fuerza se puede
expresar como la derivada de una función de energía potencial,1 y la ley de Newton dice
. ) Esto, junto con las condiciones iniciales adecuadas (normalmente la posición y
velocidad en ), determina.

Figura 1.1: Una “partícula” obligada a moverse en una dimensión bajo la influencia de una fuerza específica.

La mecánica cuántica aborda este mismo problema de manera muy diferente. En este caso lo que buscamos es la

partículafunción de onda, y lo obtenemos resolviendo el Ecuación de Schrödinger:

(1,1)

Aquí I es la raíz cuadrada de , y es la constante de Planck, o más bien, su original constanteh) dividido por :

(1,2)

La ecuación de Schrödinger juega un papel lógicamente análogo a la segunda ley de Newton: dadas las condiciones

iniciales adecuadas (típicamente,), la ecuación de Schrödinger determina para todo el tiempo futuro, al igual que, en

mecánica clásica, la ley de Newton determina para todo el tiempo futuro.2

dieciséis
1.2 La interpretación estadística

Pero que exactamente es esta "función de onda" y qué hace por ti una vez que has tiene ¿eso? Después de todo, una partícula,

por su naturaleza, está localizada en un punto, mientras que la función de onda (como sugiere su nombre) se extiende en el

espacio (es una función deX, para cualquier dado t). ¿Cómo puede un objeto así representar el estado de unpartícula? La

respuesta la proporciona Borninterpretación estadística, que dice que da la probabilidad de encontrar la partícula en el punto

X, en el momento t—O, más precisamente,3

(1,3)

La probabilidad es la zona bajo el gráfico de . Para la función de onda en la Figura1.2, serias muy probable
para encontrar la partícula en las proximidades del punto A, dónde es grande y relativamente Naciones Unidases probable que lo encuentre cerca del punto B.

Figura 1.2: Una función de onda típica. El área sombreada representa la probabilidad de encontrar la partícula entrea y B. Es

relativamente probable que la partícula se encuentre cercaA, y es poco probable que se encuentre cerca B.

La interpretación estadística introduce una especie de indeterminación a la mecánica cuántica, porque incluso si

sabe todo lo que la teoría tiene que decirle sobre la partícula (es decir, su función de onda), aún no puede predecir con

certeza el resultado de un simple experimento para medir su posición; toda la mecánica cuántica tiene que la oferta es

estadístico información sobre el posible resultados. Esta indeterminación ha sido profundamente inquietante tanto para
los físicos como para los filósofos, y es natural preguntarse si es un hecho de la naturaleza o un defecto de la teoría.

Suponga que yo hacer mido la posición de la partícula, y encuentro que está en el punto C.4 Pregunta: ¿Dónde

estaba la partícula? antes de Hice la medida? Hay tres respuestas plausibles a esta pregunta, y sirven para caracterizar

las principales escuelas de pensamiento con respecto a la indeterminación cuántica:

1. El realista posición: La partícula estaba en C. Esto ciertamente parece razonable, y es la respuesta que propugnaba Einstein. Sin embargo, tenga en cuenta que si

esto es cierto, entonces la mecánica cuántica es unaincompleto teoría, ya que la partícula realmente fue a Cy, sin embargo, la mecánica cuántica no pudo decírnoslo.

Para el realista, la indeterminación no es un hecho de la naturaleza, sino un reflejo de nuestra ignorancia. Como dijo d'Espagnat, "la posición de la partícula nunca fue

indeterminada, sino simplemente desconocida para el experimentador".5 Evidentemente no es toda la historia: alguna información adicional (conocida como variable

oculta) es necesario para proporcionar una descripción completa de la partícula.

2. El ortodoxo posición: La partícula no estaba realmente en ninguna parte. Fue el acto de medir lo que lo obligó a "tomar una posición" (aunque cómo
y por qué se decidió al respecto C no nos atrevemos a preguntar). Jordan lo dijo de la manera más cruda: “Las observaciones no solomolestar lo que se va
a medir, ellosProduce esto nosotros obligar [la partícula] para asumir una posición definida ".6 Esta vista (la llamada Interpretación de Copenhague), está
asociado con Bohr y sus seguidores. Entre los físicos, siempre ha sido la posición más aceptada. Sin embargo, tenga en cuenta que, si es correcto, hay algo
muy peculiar en el acto de medir, algo que casi un siglo de debate ha hecho muy poco por esclarecer.

17
3. El agnóstico posición: Negarse a contestar. Esto no es tan tonto como parece; después de todo, ¿qué sentido puede tener hacer afirmaciones sobre
el estado de una partícula? antes de una medida, cuando la única forma de saber si acertó es precisamente hacer una medida, en cuyo caso lo que
obtiene ya no es "antes de la medida"? Es metafísica (en el sentido peyorativo de la palabra) preocuparse por algo que, por su naturaleza, no puede ser
probado. Pauli dijo: "Uno no debería atormentarse más la cabeza sobre el problema de si algo de lo que uno no puede saber nada existe de todos modos,
que sobre la antigua pregunta de cuántos ángeles son capaces de sentarse en la punta de una aguja".7 Durante décadas, esta fue la posición "alternativa"
de la mayoría de los físicos: intentarían venderle la respuesta ortodoxa, pero si era persistente, se retirarían a la respuesta agnóstica y pondrían fin a la
conversación.

Hasta hace relativamente poco tiempo, las tres posiciones (realista, ortodoxa y agnóstica) tenían sus partidarios.

Pero en 1964 John Bell asombró a la comunidad de la física al demostrar que hace unobservable diferencia si la partícula

tenía una posición precisa (aunque desconocida) antes de la medición, o no. El descubrimiento de Bell eliminó

efectivamente el agnosticismo como una opción viable y lo convirtió en unexperimental pregunta si 1 o 2 es la elección

correcta. Volveré a esta historia al final del libro, cuando estará en una mejor posición para apreciar el argumento de

Bell; por ahora, baste decir que los experimentos han confirmado de manera decisiva la interpretación ortodoxa:8 una

partícula simplemente no tiene una posición precisa antes de la medición, al igual que las ondas en un estanque; es el

proceso de medición que insiste en un número en particular, y por lo tanto, en cierto sentidocrea el resultado específico,

limitado sólo por la ponderación estadística impuesta por la función de onda.

¿Qué pasa si hago un segundo medición, inmediatamente después de la primera? Lo conseguiríaC de


nuevo, ¿o el acto de medir arroja un número completamente nuevo cada vez? En esta pregunta, todos están de
acuerdo: una medición repetida (en la misma partícula) debe devolver el mismo valor. De hecho, sería difícil
demostrar que la partícula se encontró realmente enC en primera instancia, si esto no pudiera confirmarse
mediante la repetición inmediata de la medición. ¿Cómo explica la interpretación ortodoxa el hecho de que la
segunda medición está destinada a producir el valorC? Debe ser que la primera medición altera radicalmente la
función de onda, de modo que ahora tiene un pico pronunciado sobreC (Figura 1.3). Decimos que la función de
ondacolapsa, al medir, a un pico en el punto C (pronto se extiende de nuevo, de acuerdo con la ecuación de
Schrödinger, por lo que la segunda medición debe realizarse rápidamente). Hay, entonces, dos tipos de procesos
físicos completamente distintos: los "ordinarios", en los que la función de onda evoluciona pausadamente bajo la
ecuación de Schrödinger, y las "medidas", en las que colapsa repentina y discontinuamente.9

Figura 1.3: Colapso de la función de onda: gráfico de inmediatamente después una medición ha encontrado el

partícula en el punto C.

Ejemplo 1.1
Interferencia de electrones. He afirmado que las partículas (electrones, por ejemplo) tienen una naturaleza

ondulatoria, codificada. ¿Cómo podemos comprobar esto en el laboratorio?

La firma clásica de un fenómeno ondulatorio es interferencia: dos olas en fase interfieren constructivamente, y

fuera de fase interfieren destructivamente. La naturaleza ondulatoria de la luz se confirmó en

18
1801 por el famoso experimento de doble rendija de Young, que muestra "franjas" de interferencia en una pantalla distante

cuando un rayo monocromático pasa a través de dos rendijas. Si esencialmente se hace el mismo experimento conelectrones,

se desarrolla el mismo patrón,10 confirmando la naturaleza ondulatoria de los electrones.

Supongamos ahora que disminuimos la intensidad del haz de electrones, hasta que solo haya un electrón

presente en el aparato en un momento determinado. Según la interpretación estadística, cada electrón producirá un

punto en la pantalla. La mecánica cuántica no puede predecir la precisiónlocalización de ese lugar, todo lo que puede

decirnos es el probabilidad de un electrón dado que aterriza en un lugar particular. Pero si somos pacientes y

esperamos cien mil electrones, uno a la vez, para hacer el viaje, los puntos acumulados revelan el patrón clásico de

interferencia de dos rendijas (Figura1.4). 11

Figura 1.4: Acumulación del patrón de interferencia de electrones. (a) Ocho electrones, (b) 270 electrones, (c) 2000

electrones, (d) 160 000 electrones. Reimpreso por cortesía del Laboratorio Central de Investigación, Hitachi, Ltd.,

Japón.

Por supuesto, si cierra una rendija, o de alguna manera se las arregla para detectar por qué rendija pasa cada

electrón, el patrón de interferencia desaparece; la función de onda de la partícula emergente ahora es

completamente diferente (en el primer caso porque las condiciones de contorno para la ecuación de

Schrödinger han cambiado, y en el segundo debido al colapso de la función de onda en la medición). Pero con

ambas rendijas abiertas y sin interrupción del electrón en vuelo, cada electrón interfiere consigo mismo; no

pasó por una rendija ni por la otra, sino por las dos a la vez, al igual que una ola de agua que choca contra un

muelle con dos aberturas se interfiere consigo misma. No hay nada de misterioso en esto, una vez que haya

aceptado la noción de que las partículas obedecen a una ecuación de onda. La verdadasombroso La cosa es el

ensamblaje blip-by-blip del patrón. En cualquier teoría de ondas clásica, el patrón se desarrollaría de manera

suave y continua, simplemente volviéndose más intenso a medida que pasa el tiempo. El proceso cuántico se

parece más a la pintura puntillista de Seurat: la imagen surge de las contribuciones acumulativas de todos los

puntos individuales.12

19
20
1.3 Probabilidad

21
1.3.1 Variables discretas

Debido a la interpretación estadística, la probabilidad juega un papel central en la mecánica cuántica, por lo que ahora

me desvío para una breve discusión de la teoría de la probabilidad. Se trata principalmente de introducir algo de

notación y terminología, y lo haré en el contexto de un ejemplo sencillo.

Imagine una habitación con catorce personas, cuyas edades son las siguientes:

una persona de 14 años, una

persona de 15 años, tres

personas de 16 años, dos

personas de 22 años, dos

personas de 24 años, cinco

personas de 25 años.

Si dejamos representar el número de personas mayores de edad j, luego

tiempo , por ejemplo, es cero. lostotal el número de personas en la habitación es

(1,4)

(En el ejemplo, por supuesto, . ) Figura1,5 es un histograma de los datos. Las siguientes son algunas preguntas

uno podría preguntar acerca de esta distribución.

Figura 1.5: Histograma que muestra el número de personas, , con edad j, por ejemplo en la sección 1.3.1.

Pregunta 1 Si seleccionó a una persona al azar de este grupo, ¿cuál es la probabilidad que la edad de esta
persona sería de 15?
Respuesta Una posibilidad en 14, ya que hay 14 opciones posibles, todas igualmente probables, de
las cuales solo una tiene esa edad en particular. Si es la probabilidad de envejecerj, luego
, etcétera. En general,

22
(1,5)

Observe que la probabilidad de obtener cualquiera 14 o 15 es el suma de las probabilidades individuales (en este caso, 1/7). En

particular, la suma detodos las probabilidades son 1: la persona que seleccione debe tener algunos la edad:

(1,6)

Pregunta 2 Cuál es el Más probable ¿la edad?

Respuesta 25, obviamente; cinco personas comparten esta edad, mientras que como máximo tres tienen cualquier otra edad.

Lo mas probablej es el j para lo cual es un máximo.Pregunta 3 Cuál es el mediana ¿la edad?

Respuesta 23, para 7 personas son menores de 23 y 7 son mayores. (La mediana es ese valor dej tal que la probabilidad de

obtener un resultado mayor es la misma que la probabilidad de obtener un resultado menor).

Pregunta 4 Cuál es el promedio (o significar) ¿la edad?

Respuesta

En general, el valor medio de j (que escribiremos así: ) es

(1,7)

Observe que no es necesario que haya nadie con la edad promedio o la edad mediana; en este ejemplo, nadie
tiene 21 o 23 años. En mecánica cuántica, el promedio suele ser la cantidad de interés; en ese contexto ha
llegado a ser llamado elvalor esperado. Es un término engañoso, ya que sugiere que este es el resultado que
probablemente obtendría si hiciera una sola medición (ese sería el valor más probable, no el valor promedio),
pero me temo que estamos atrapados con él.

Pregunta 5 ¿Cuál es el promedio de cuadrícula de las edades?


Respuesta Podría obtener, con probabilidad 1/14, o , con probabilidad 1/14, o
, con probabilidad 3/14, y así sucesivamente. El promedio, entonces, es

(1,8)

En general, el valor medio de algunos función de j es dado por

(1,9)

(Ecuaciones 1,6, 1,7, y 1.8 son, si lo desea, casos especiales de esta fórmula). Tener cuidado: El promedio de los cuadrados,

, es no igual, en general, al cuadrado de la media,. Por ejemplo, si la habitación contiene solo dos
bebés, de 1 y 3 años, entonces, pero.

23
Ahora bien, hay una diferencia notable entre los dos histogramas de la Figura 1,6, a pesar de que tienen la misma

mediana, el mismo promedio, el mismo valor más probable y el mismo número de elementos: el primero tiene un pico

pronunciado sobre el valor promedio, mientras que el segundo es amplio y plano. (El primero podría representar el perfil de

edad de los estudiantes en un aula de una gran ciudad, el segundo, tal vez, una escuela rural de una sola habitación.)

Necesitamos una medida numérica de la cantidad de "propagación" en una distribución, con respecto a la promedio. La forma

más obvia de hacer esto sería averiguar qué tan lejos está cada individuo del promedio,

(1,10)

y calcular el promedio de . El problema es, por supuesto, que obtienescero:

(Tenga en cuenta que es constante, no cambia a medida que pasa de un miembro de la muestra a otro, por lo que puede

debe tomarse fuera de la suma.) Para evitar este irritante problema, puede decidir promediar el valor absoluto de . Pero

es desagradable trabajar con valores absolutos; en cambio, solucionamos el problema de las señalescuadraturaantes de

promediar:

(1,11)

Esta cantidad se conoce como diferencia de la distribución; σ en sí (la raíz cuadrada del promedio del cuadrado
de la desviación del promedio, ¡trago!) se llamaDesviación Estándar. Esta última es la medida habitual de la
propagación.

Figura 1.6: Dos histogramas con la misma mediana, el mismo promedio y el mismo valor más probable, pero diferentes

desviaciones estándar.

Hay un pequeño teorema útil sobre las varianzas:

Tomando la raíz cuadrada, la desviación estándar en sí se puede escribir como

24
(1,12)

En la práctica, esta es una forma mucho más rápida de obtener σ que mediante la aplicación directa de la Ecuación 1,11:

simplemente calcula y, resta y saca la raíz cuadrada. Por cierto, te advertí hace un momento que no es, en general, igual

a. Dado que es claramente no negativo (de su definición1,11), Ecuación 1.12 implica que

(1,13)

y los dos son iguales solo cuando , es decir, para distribuciones sin ningún diferencial (cada miembro
tienen el mismo valor).

25
1.3.2 Variables continuas

Hasta ahora, he asumido que estamos lidiando con una discreto variable, es decir, una que puede tomar solo ciertos valores

aislados (en el ejemplo, j tenía que ser un número entero, ya que di las edades solo en años). Pero es bastante simple

generalizar acontinuo distribuciones. Si selecciono a una persona al azar de la calle, la probabilidad de que su edad sea

precisamente 16 años, 4 horas, 27 minutos y 3.333 ... segundos es cero. Lo único sensato de lo que hablar es la probabilidad de

que su edad se encuentre en algúnintervalo—Digamos, entre 16 y 17. Si el intervalo es suficientemente corto, esta probabilidad

es proporcional a la longitud del intervalo. Por ejemplo, la probabilidad de que su edad esté entre 16 y 16 años másdos días es

presumiblemente el doble de probabilidad de que esté entre 16 y 16 más uno día. (A menos que, supongo, hubo un

extraordinario baby boom hace 16 años, exactamente ese día, en cuyo caso simplemente hemos elegido un intervalo

demasiado largo para que se aplique la regla. Si el baby boom duró seis horas, tomaremos intervalos de un segundo o menos,

para estar seguros. Técnicamente, estamos hablando deinfinitesimal intervalos.) Así

(1,14)

El factor de proporcionalidad, , a menudo se llama vagamente "la probabilidad de obtener X, "Pero esto es descuidado

idioma; un término mejor esdensidad de probabilidad. La probabilidad de queX entre mentiras a y B (a finito intervalo) viene

dada por la integral de:

(1,15)

y las reglas que dedujimos para distribuciones discretas se traducen de la manera obvia:

(1,16)

(1,17)

(1,18)

(1,19)

Ejemplo 1.2
Suponga que alguien deja caer una piedra de un acantilado de altura h. Mientras cae, tomo un millón de

fotografías, a intervalos aleatorios. En cada imagen mido la distancia que ha caído la roca.Pregunta: Cuál es el

promedio de todas estas distancias? Es decir, ¿cuál es elpromedio de tiempo de la distancia recorrida?13
Solución: La roca comienza en reposo y gana velocidad a medida que cae; pasa más tiempo cerca de la cima, por lo que la

distancia promedio seguramente será menor que. Ignorando la resistencia del aire, la distanciaX en el momento t es

26
La velocidad es , y el tiempo total de vuelo es . La probabilidad de que un

se tomó una fotografía particular entre t y es , por lo que la probabilidad de que muestre un

distancia en el rango correspondiente X para es

Por tanto, la probabilidad densidad (Ecuación 1,14) es

(fuera de este rango, por supuesto, la densidad de probabilidad es cero).

Podemos verificar este resultado, usando la Ecuación 1,16:

los promedio distancia (Ecuación 1,17) es

que es algo menos que , como se esperaba.

Figura 1,7 muestra la gráfica de . Observe que una probabilidaddensidad aunque puede ser infinito

probabilidad en sí misma (la integral de ρ) debe ser, por supuesto, finito (de hecho, menor o igual a 1).

Figura 1.7: La densidad de probabilidad en el ejemplo 1.2: .

∗ Problema 1.1 Para la distribución de edades en el ejemplo de la sección 1.3.1:

(a) Calcular y .
(B) Determinar para cada jy usa la ecuación 1,11 para calcular el estándar
desviación.

(C) Use sus resultados en (a) y (b) para verificar la Ecuación 1.12.

27
Problema 1.2

(a) Encuentre la desviación estándar de la distribución en el ejemplo 1.2.

(B) ¿Cuál es la probabilidad de que una fotografía, seleccionada al azar,


muestre una distancia X más de una desviación estándar del
promedio?

∗ Problema 1.3 Considera el gaussiano distribución

dónde A, a, y dentro deson constantes reales positivas. (Las integrales necesarias son

la contraportada.)

(a) Usar ecuación 1,16 para determinar A.

(B) Encuentre, y σ.
(C) Dibuja la gráfica de .

28
1.4 Normalización
Volvamos ahora a la interpretación estadística de la función de onda (Ecuación 1.3), que dice que
es la densidad de probabilidad para encontrar la partícula en el punto X, en el momento t. Sigue (Ecuación1,16)

que la integral de más todo x debe ser 1 (la partícula debe ser algunosdónde):

(1,20)

Sin esto, la interpretación estadística sería una tontería.


Sin embargo, este requisito debería molestarlo: después de todo, se supone que la función de onda está

determinada por la ecuación de Schrödinger; no podemos imponer una condición extraña sin verificar que las dos sean

consistentes. Bueno, un vistazo a la ecuación1.1 revela que si es una solución, también lo es, donde A es cualquier

constante (compleja). Lo que debemos hacer, entonces, es elegir este factor multiplicativo indeterminado para

asegurarnos de que la ecuación1,20 Está satisfecho. Este proceso se llamanormalizando la función de onda. Para

algunas soluciones a la ecuación de Schrödinger, la integral esinfinito; en ese casono el factor multiplicativo lo convertirá

en 1. Lo mismo ocurre con la solución trivial. Talno normalizable las soluciones no pueden representar partículas y

deben ser rechazadas. Los estados físicamente realizables corresponden a lacuadrado-integrable soluciones a la

ecuación de Schrödinger.14

¡Pero espere un minuto! Supongamos que he normalizado la función de onda en el momento . Como se que es
voluntad Quédate normalizado, a medida que pasa el tiempo y evoluciona? (No puedes mantenerrenormalizando la

función de onda, para entonces A se convierte en una función de t, y ya no tienes una solución para la ecuación de

Schrödinger). Afortunadamente, la ecuación de Schrödinger tiene la propiedad notable de que conserva

automáticamente la normalización de la función de onda; sin esta característica crucial, la ecuación de Schrödinger sería

incompatible con la interpretación estadística, y toda la teoría se derrumbaría.

Esto es importante, así que será mejor que hagamos una pausa para una prueba cuidadosa. Para empezar,

(1,21)

(Tenga en cuenta que el integral es una función solo de t, entonces uso un total derivado a la izquierda, pero el integrando es

una función de X al igual que t, entonces es un parcial derivada a la derecha.) Por la regla del producto,

(1,22)

Ahora, la ecuación de Schrödinger dice que

(1,23)

y por lo tanto también (tomando el conjugado complejo de la ecuación 1,23)

(1,24)

29
asi que

(1,25)

La integral en la ecuación 1,21 ahora se puede evaluar explícitamente:

(1,26)

Pero debe ir a cero como X va a infinito; de lo contrario, la función de onda no sería


normalizable.15 Resulta que

(1,27)

y por tanto que la integral es constante (independiente del tiempo); si para está normalizado en , eso corsé normalizado

todo el tiempo futuro. QED

Problema 1.4 En el momento una partícula está representada por la función de onda

dónde A, a, y B son constantes (positivas).

(a) Normalizar (es decir, encontrar A, en términos de a y B).

(B) Bosquejo , como una función de X.

(C) ¿Dónde es más probable que se encuentre la partícula, en ?


(D) ¿Cuál es la probabilidad de encontrar la partícula a la izquierda de a?
Compruebe su resultado en los casos límite y.
(mi) ¿Cuál es el valor esperado de X?

∗ Problema 1.5 Considere la función de onda

dónde A, y ω son constantes reales positivas. (Veremos en el capítulo2 para que

potencialV) esta función de onda satisface la ecuación de Schrödinger.)

(a) Normalizar.
(B) Determine los valores esperados de X y .
(C) Encuentre la desviación estándar de X. Dibuja la gráfica de , como una función

de X, y marque los puntos y, para ilustrar el sentido en el que σ


representa la "extensión" en X. ¿Cuál es la probabilidad de que la
partícula se encuentre fuera de este rango?

30
31
1.5 Momento
Para una partícula en estado, el valor esperado de X es

(1,28)

¿Qué significa esto exactamente? Enfáticamente lo haceno significa que si mide la posición de una partícula una y
otra vez, es el promedio de los resultados que obtendrá. Por el contrario: la primera medición (cuyo resultado es
indeterminado) colapsará la función de onda a un pico en el valor realmente obtenido, y las mediciones
posteriores (si se realizan rápidamente) simplemente repetirán el mismo resultado. Más bien, ¿es el promedio de
las mediciones realizadas en partículastodo en el estado , lo que significa que debe encontrar alguna forma de
devolver la partícula a su estado original después de cada medición, o debe preparar un conjunto conjunto de
partículas, cada una en el mismo estado, y medir las posiciones de todas ellas:
es el promedio de estas resultados. Me gusta imaginar una fila de botellas en un estante, cada una de las cuales contiene

una partícula en el estado (en relación con el centro de la botella). A cada botella se le asigna un estudiante graduado con una

regla y, a una señal, todos miden las posiciones de sus respectivas partículas. Luego construimos un histograma de los

resultados, que debería coincidir, y calculamos el promedio, que debería coincidir. (Por supuesto, dado que solo estamos

usando una muestra finita, no podemos esperar una concordancia perfecta, pero cuantas más botellas usemos, más cerca

deberíamos estar). En resumen,el valor esperado es el promedio de mediciones en un conjunto de sistemas preparados de

manera idéntica,no el promedio de mediciones repetidas en un mismo sistema.

Ahora, a medida que pasa el tiempo, cambiará (debido a la dependencia del tiempo del interesado),en
y podríamos ser
saber qué tan rápido se mueve. Refiriéndose a las ecuaciones 1,25 y 1,28, vemos esodieciséis

(1,29)

Esta expresión se puede simplificar mediante la integración por partes:17

(1,30)

(Utilicé el hecho de que , y descartó el término límite, en el terreno que va a cero en


infinito.) Realizando otra integración por partes, en el segundo término, concluimos:

(1,31)

¿Qué vamos a hacer con este resultado? Tenga en cuenta que estamos hablando de la "velocidad" de laexpectativa

valor deX, que no es lo mismo que la velocidad del partícula. Nada de lo que hemos visto hasta ahora nos permitiría

calcular la velocidad de una partícula. Ni siquiera está claro qué velocidadmedio en mecánica cuántica: si la partícula no

tiene una posición determinada (antes de la medición), tampoco tiene una velocidad bien definida. Todo lo que

podemos pedir razonablemente esprobabilidad de obtener un valor particular. Veremos en el capítulo3 cómo construir

la densidad de probabilidad para la velocidad, dada; por el momento bastará con postular que elEl valor esperado de la

velocidad es igual a la derivada del tiempo del valor esperado de la posición.:

32
(1,32)

Ecuación 1,31 nos dice, entonces, cómo calcular directamente desde .

En realidad, es habitual trabajar con impulso , en lugar de velocidad:

(1,33)

Déjame escribir las expresiones para y de una manera más sugerente:

(1,34)

(1,35)

Decimos que el operador18 X "representa "posición, y el operador calcula los "Representa" el impulso;
valores esperados" intercalamos "al operador apropiado entre ay, e integrar.
Eso es lindo, pero ¿qué pasa con otras cantidades? El hecho es,todos Las variables dinámicas clásicas se pueden expresar

en términos de posición y momento. La energía cinética, por ejemplo, es

y el momento angular es

(esto último, por supuesto, no ocurre para el movimiento en una dimensión). Para calcular el valor
esperado dealgunatal cantidad, simplemente reemplazamos cada pag por, inserte el operador resultante
entre y, e integre:

(1,36)

Por ejemplo, el valor esperado de la energía cinética es

(1,37)

Ecuación 1,36 es una receta para calcular el valor esperado de cualquier cantidad dinámica, para una partícula en estado;

subsume ecuaciones1,34 y 1,35 como casos especiales. He intentado hacer la ecuación1,36 parece plausible, dada la

interpretación estadística de Born, pero en realidad esto representa una forma tan radicalmente nueva de hacer negocios (en

comparación con la mecánica clásica) que es una buena idea obtener algo de práctica utilizando antes de que regresemos (en el

capítulo 3) y ponerlo sobre una base teórica más firme. Mientras tanto, si prefiere pensar en ello como unaxioma, eso está bien

para mí.

33
Problema 1.6 ¿Por qué no puede hacer la integración por partes directamente en la expresión

del medio en la ecuación? 1,29: Arrastre la derivada del tiempo hacia X, tenga en cuenta que

y concluir eso?

∗ Problema 1.7 Calcular . Respuesta:

(1,38)

Esta es una instancia de Teorema de Ehrenfest, que afirma que Los valores de expectativa obedecen a

las leyes clásicas.19

Problema 1.8 Suponga que agrega una constante a la energía potencial (por "constante"

Me refiero independiente de X al igual que t). Enclásico mecánica, esto no cambia


nada, pero ¿qué pasa con cuántico ¿mecánica? Muestre que la función de onda toma
un factor de fase dependiente del tiempo:. ¿Qué efecto tiene esto en el valor
esperado de una variable dinámica?

34
1.6 El principio de incertidumbre

Imagina que estás sosteniendo un extremo de una cuerda muy larga y generas una ola agitándola hacia arriba y hacia abajo

rítmicamente (Figura 1.8). Si alguien le pregunta "¿Dónde exactamentees ¿esa ola?" probablemente pensarías que estaba un

poco loco: la ola no es precisamentealgunadonde: se extiende a más de 50 pies aproximadamente. Por otro lado, si te

preguntara qu eslongitud de onda es decir, podría darle una respuesta razonable: parece como unos 6 pies. Por el contrario, si

le dio a la cuerda un tirón repentino (Figura1,9), obtendría una protuberancia relativamente estrecha viajando por la línea. Esta

vez, la primera pregunta (¿Dónde está exactamente la onda?) Es sensata, y la segunda (¿Cuál es su longitud de onda?) Parece

una locura; ni siquiera es vagamente periódica, entonces, ¿cómo se le puede asignar una longitud de onda? Por supuesto,

puede dibujar casos intermedios, en los que la onda esequitativamente bien localizado y la longitud de onda es equitativamente

bien definida, pero hay una compensación ineludible aquí: cuanto más precisa es la posición de una onda, menos precisa es su

longitud de onda, y viceversa.20 Un teorema en el análisis de Fourier hace que todo esto sea riguroso, pero por el momento sólo

me interesa el argumento cualitativo.

Figura 1.8: Una ola con un (bastante) bien definido longitud de onda, pero un mal definido posición.

Figura 1.9: Una ola con un (bastante) bien definido posición, pero un mal definido longitud de onda.

Esto se aplica, por supuesto, a alguna fenómeno ondulatorio y, por tanto, en particular a la función de onda de la

mecánica cuántica. Pero la longitud de onda de está relacionada con laimpulso de la partícula por el fórmula de Broglie

:21

(1,39)

Por lo tanto, una extensión en longitud de onda corresponde a un spread en impulso, y nuestra observación general ahora dice que cuanto más

precisamente determinada está la posición de una partícula, menos precisamente es su momento. Cuantitativamente,

(1,40)

dónde es la desviación estándar en X, y es la desviación estándar en pag. Este es el famoso de Heisenberg

principio de incertidumbre. (Lo probaremos en el capítulo3, pero quería mencionarlo de inmediato, para que pueda probarlo

en los ejemplos del capítulo 2.)

Por favor, comprenda cuál es el principio de incertidumbre. medio: Al igual que las mediciones de posición, las mediciones de la

cantidad de movimiento dan respuestas precisas; la "extensión" aquí se refiere al hecho de que las mediciones realizadas en sistemas

preparados de manera idéntica no producen resultados idénticos. Puede, si lo desea, construir un estado tal que

35
Las mediciones de posición estarán muy juntas (al hacer un "pico" localizado), pero pagará un precio:
Las mediciones de la cantidad de movimiento en este estado estarán muy dispersas. O puede preparar un estado con

un impulso definido (haciendo una onda sinusoidal larga), pero en ese caso, las mediciones de posición estarán muy

dispersas. Y, por supuesto, si estás de muy mal humor puedes crear un estado para el que ni la posición ni el impulso

están bien definidos: Ecuación1,40 es un desigualdady no hay límite sobre cómo grande y puede ser
simplemente haga una línea larga y ondulada con muchos baches y baches y sin una estructura periódica.

∗ Problema 1.9 Una partícula de masa metro tiene la función de onda

dónde A y a son constantes reales positivas.

(a) Encontrar A.

(B) ¿Para qué función de energía potencial, la , ¿es esta una solución al
ecuación de Schrödinger?

(C) Calcule los valores esperados de ,y.


(D) Encontrar y . ¿Su producto es consistente con el principio de
incertidumbre?

36
Más problemas en el capítulo 1

Problema 1.10 Considere los primeros 25 dígitos en la expansión decimal de π (3, 1,

4, 1, 5, 9,…).

(a) Si seleccionó un número al azar, de este conjunto, ¿cuáles son las

probabilidades de obtener cada uno de los 10 dígitos?

(B) ¿Cuál es el dígito más probable? ¿Cuál es el dígito mediano? Cual es el


valor medio?
(C) Encuentra la desviación estándar de esta distribución.

Problema 1.11 [Este problema se generaliza Ejemplo 1.2.] Imagina una partícula de masa

metro y energia mi en un pozo potencial, deslizándose sin fricción hacia adelante y hacia

atrás entre los puntos de inflexión clásicos (a y B En figura 1,10). Clásicamente, la

probabilidad de encontrar la partícula en el rangodx (si, por ejemplo, tomó una instantánea

en un momento aleatorio t) es igual a la fracción del tiempo T se necesita para llegar de a

para B que pasa en el intervalo dx:

(1,41)

dónde es la velocidad, y

(1,42)

Por lo tanto

(1,43)

Este es quizás el análogo clásico más cercano22 para .


(a) Utilice la conservación de la energía para expresar en términos de mi y .
(B) Como ejemplo, encuentre para el oscilador armónico simple,
. Trama y comprobar que esté correctamente normalizado.

(C) Para el oscilador armónico clásico del inciso b), encuentre ,,y.

37
Figura 1.10: Partícula clásica en un pozo potencial.

∗∗ Problema 1.12 ¿Y si estuviéramos interesados en la distribución de Momentos


, para el oscilador armónico clásico (Problema 1,11(B)).
(a) Encuentra la distribución de probabilidad clásica (tenga en cuenta que pag rangos desde

para ).
(B) Calcular, ,y.
(C) Cual es el clásico producto de incertidumbre, , para este sistema? Aviso
que este producto puede ser tan pequeño como quieras, clásicamente, simplemente enviando

. Pero en la mecánica cuántica, como veremos en el capítulo2, los


La energía de un oscilador armónico simple no puede ser menor que, donde

es la frecuencia clásica. En ese caso, ¿qué puedes decir sobre


el producto ?

Problema 1.13 Verifique sus resultados en Problema 1,11(b) con lo siguiente


"Experimento numérico". La posición del oscilador en el momentot es

(1,44 )

Bien podrías tomar (que establece la escala de tiempo) y (ese


establece la escala de longitud). Haz un diagrama deX a 10,000 veces al azar,
y compárelo con.
Insinuación: En Mathematica, primero defina

luego construya una tabla de posiciones:

y finalmente, haga un histograma de los datos:

Mientras tanto, haga un gráfico de la función de densidad, , y, usando Show,


superponga los dos.

Problema 1.14 Dejar ser la probabilidad de encontrar la partícula en el rango

, en el momento t.

(a) Muestra esa

dónde

38
Cuales son las unidades de ? Comentario: J se llama el probabilidad
Actual, porque le indica la velocidad a la que la probabilidad "fluye" más allá del

punto X. Si está aumentando, entonces fluye más probabilidad hacia la región en

un extremo que hacia afuera en el otro.

(B) Encuentre la corriente de probabilidad para la función de onda en el problema 1,9. (Me temo

que este no es un ejemplo muy conciso; encontraremos otros más sustanciales a su debido

tiempo).

Problema 1.15 Muestra esa

para dos soluciones (normalizables) cualesquiera de la ecuación de Schrödinger


(con la misma), y.

Problema 1.16 Se representa una partícula (en el momento ) por la función de onda

(a) Determine la constante de normalización A.

(B) ¿Cuál es el valor esperado de X?


(C) ¿Cuál es el valor esperado de pag? (Tenga en cuenta que ustedno poder obtenerlo de

. ¿Por qué no?)

(D) Encuentre el valor esperado de .


(mi) Encuentre el valor esperado de .
(F) Encuentra la incertidumbre en .
(gramo) Encuentra la incertidumbre en .
(h) Verifique que sus resultados sean consistentes con el principio de incertidumbre.

∗∗ Problema 1.17 Suponga que quisiera describir un partícula inestable, ese


se desintegra espontáneamente con una "vida" τ. En ese caso, la probabilidad total de

encontrar la partícula en algún lugar deberíano ser constante, pero debería disminuir

a (digamos) una tasa exponencial:

Una forma burda de lograr este resultado es la siguiente. En ecuación1,24 asumimos

tácitamente que V (la energía potencial) es verdadero. Eso es ciertamente razonable, pero

conduce a la "conservación de la probabilidad" consagrada en la Ecuación1,27. ¿Y si

asignamos aV una parte imaginaria:

dónde es la verdadera energía potencial y Γ es una constante real positiva?

(a) Demuestre que (en lugar de la ecuación 1,27) ahora tenemos

39
(B) Resolver y encuentre la vida útil de la partícula en términos de Γ.

Problema 1.18 En términos muy generales, la mecánica cuántica es relevante cuando el

La longitud de onda de Broglie de la partícula en cuestión es mayor que el


tamaño característico del sistema. En equilibrio térmico a temperatura (Kelvin)T,
la energía cinética promedio de una partícula es

(dónde es la constante de Boltzmann), por lo que la longitud de onda típica de De Broglie es

(1,45)

El propósito de este problema es determinar qué sistemas tendrán que ser tratados

mecánicamente cuánticamente y cuáles pueden describirse de manera clásica con seguridad.

(a) Sólidos. El espaciado de celosía en un sólido típico es de alrededor de nm. Encuentre la

temperatura por debajo de la cual el23 electrones en un sólido son mecánicas

cuánticas. ¿Por debajo de qué temperatura están losnúcleos en una mecánica cuántica

sólida? (Utilice el silicio como ejemplo).

Moral: Los electrones libres en un sólido son siempre mecánica cuántica; los núcleos

son generalmenteno mecánica cuántica. Lo mismo ocurre con los líquidos (para los

que el espaciado interatómico es aproximadamente el mismo), con la excepción del

helio por debajo de 4 K.

(b) Gases. ¿A qué temperaturas están los átomos en un gas ideal a presión? PAG

¿mecánica cuántica? Insinuación: Utilice la ley de los gases ideales para deducir

el espaciamiento interatómico.

Respuesta: . Obviamente (para que el gas se vea

comportamiento cuántico) que queremos metro ser como pequeña como sea posible, y PAG

como grande como sea posible. Ingrese los números del helio a presión atmosférica. ¿Es el

hidrógeno en el espacio exterior (donde el espacio interatómico es de aproximadamente 1

cm y la temperatura es de 3 K) mecánica cuántica? (Suponga que es hidrógeno

monoatómico, no H.)

1
Las fuerzas magnéticas son una excepción, pero no nos preocupemos por ellas todavía. Por cierto, asumiremos a lo largo de este libro que el movimiento

no es relativista. .
2
Para un delicioso relato de primera mano de los orígenes de la ecuación de Schrödinger, consulte el artículo de Felix Bloch en Física hoy, Diciembre de 1976.

3
La función de onda en sí es compleja, pero la (dónde es el complejo conjugado de ) es real y no negativo, como
probabilidad, por supuesto, debe ser.
4
Por supuesto, ningún instrumento de medición es perfectamente preciso; lo que yosignificar es que la partícula fue encontrada en la vecindad de C, según lo

definido por la precisión del equipo.


5
Bernard d'Espagnat, "La teoría cuántica y la realidad" (Científico americano, Noviembre de 1979, pág. 165).
6
Citado en un hermoso artículo de N. David Mermin, "¿Está la luna allí cuando nadie mira?" (Física hoy, Abril de 1985, pág. 38).

40
7
Ibíd., Pág. 40.
8
Esta afirmación es demasiado fuerte: existen teorías viables de variables ocultas no locales (en particular la de David Bohm) y otras formulaciones (como la muchos

mundos interpretación) que no encajan claramente en ninguna de mis tres categorías. Pero creo que es prudente, al menos desde el punto de vista pedagógico,

adoptar una plataforma clara y coherente en esta etapa y preocuparse por las alternativas más adelante.
9
El papel de la medición en la mecánica cuántica es tan crítico y tan extraño que es posible que se pregunte qué es exactamente que constituye una

medida. Volveré a este tema espinoso en el Epílogo; por el momento, adoptemos la visión ingenua: una medición es el tipo de cosa que hace un científico

con bata blanca en el laboratorio, con reglas, cronómetros, contadores Geiger, etc.
10
Debido a que la longitud de onda de los electrones suele ser muy pequeña, las rendijas deben estar muy juntas. Históricamente, esto fue logrado por primera vez por

Davisson y Germer, en 1925, usando las capas atómicas en un cristal como "rendijas". Para un relato interesante, vea RK Gehrenbeck,Física hoy, Enero de 1978, página

34.
11
Ver Tonomura et al., Revista estadounidense de física, Volumen 57, Número 2, págs. 117-120 (1989), y el asombroso video asociado enwww.hitachi.com/rd/

portal/highlight/quantum/doubleslit/. Este experimento ahora se puede hacer con partículas mucho más masivas, incluidas las “bolas de Bucky”; ver M.

Arndt, et al.,Naturaleza 40, 680 (1999). Por cierto, se puede hacer lo mismo con la luz: baje la intensidad tanto que sólo esté presente un “fotón” a la vez y

obtendrá un ensamblaje idéntico punto por punto del patrón de interferencia. Ver RS Aspden,

MJ Padgett y GC Spalding, Soy. J. Phys.84, 671 (2016).


12
Creo que es importante distinguir cosas como la interferencia y la difracción que serían válidas para cualquier teoría de ondas de las características

exclusivamente mecánicas cuánticas del proceso de medición, que se derivan de la interpretación estadística.
13
Un estadístico se quejará de que estoy confundiendo el promedio de un muestra finita (un millón, en este caso) con el promedio "verdadero" (en todo el continuo). Esto

puede ser un problema incómodo para el experimentador, especialmente cuando el tamaño de la muestra es pequeño, pero aquí solo me preocupan loscierto promedio,

al cual el promedio de la muestra es presumiblemente una buena aproximación.


14
Evidentemente debe ir a cero más rápido que, como. Por cierto, la normalización solo corrige elmódulo de A; losfasepermanece
indeterminado. Sin embargo, como veremos, este último no tiene ningún significado físico de todos modos.
15
Un matemático competente puede proporcionarle contraejemplos patológicos, pero no surgen en la física; para nosotros la función de onda y todas sus

derivadas van a cero en el infinito.


dieciséis
Para evitar que las cosas se abarroten demasiado, suprimiré los límites de la integración. La .
17
regla del producto dice que

de lo que se sigue que

Debajo del signo integral, entonces, puede quitar una derivada de un factor en un producto y colocarlo sobre el otro; le costará un signo menos
y obtendrá un término límite.
18
Un "operador" es una instrucción para hacer algo a la función que sigue; toma una función y escupe alguna otra función. El operador de
posición le dice quemultiplicar por X; el operador de impulso te dice quediferenciar con respecto a X (y multiplicar el resultado por
).
19
Algunos autores limitan el término al par de ecuaciones y
20
Es por eso que un flautista debe estar en el terreno de juego, mientras que un contrabajista puede permitirse usar guantes de jardín. Para el flautín, una sesenta y

cuatro notas contiene muchos ciclos completos, y la frecuencia (ahora estamos trabajando en el dominio del tiempo, en lugar del espacio) está bien definida, mientras

que para el bajo, en un registro mucho más bajo, la sesenta y cuatro nota contiene sólo unos pocos ciclos, y todo lo que oye es una especie de "empuje" general, sin un

tono muy claro.


21
Explicaré esto a su debido tiempo. Muchos autores toman la fórmula de De Broglie como unaxioma, de lo que luego deducen la asociación de la

cantidad de movimiento con el operador. Aunque este es un enfoque conceptualmente más limpio, implica desviar las complicaciones matemáticas

que preferiría guardar para más adelante.


22
Si quieres, en lugar de fotos de uno sistema en momentos aleatorios, imagine un conjunto de tales sistemas, todos con la misma energía pero con

posiciones iniciales aleatorias, y fotografíelos todos en el mismo tiempo. El análisis es idéntico, pero esta interpretación se acerca más a la noción cuántica

de indeterminación.
23
En un sólido, los electrones internos están unidos a un núcleo particular, y para ellos el tamaño relevante sería el radio del átomo. Pero los electrones más

externos no están unidos, y para ellos la distancia relevante es el espaciado de la red. Este problema pertenece alexterior electrones.

41
2
Ecuación de Schrödinger independiente del tiempo

42
2.1 Estados estacionarios

En el capítulo 1 hablamos mucho sobre la función de onda y cómo se usa para calcular varias cantidades de interés. Ha

llegado el momento de dejar de procrastinar y afrontar lo que es, lógicamente, la pregunta previa: ¿cómoobtener

en el primero ¿lugar? Necesitamos resolver la ecuación de Schrödinger,

(2,1)

para un potencial especificado1 . En este capítulo (y la mayor parte de este libro) asumiré queV es independiente

de t. En ese caso, la ecuación de Schrödinger se puede resolver mediante el método deseparación de variables (la primera

línea de ataque del físico sobre cualquier ecuación diferencial parcial): buscamos soluciones que sean productos,

(2,2)

dónde (más bajo-case) es una función de X solo, y es una función de t solo. A primera vista, esto es un absurdo

restricción, y no podemos esperar obtener más que un pequeño subconjunto de todas las soluciones de esta manera.

Pero espera, porque las soluciones quehacer conseguir resultar de gran interés. Además (como suele ser el caso con la

separación de variables) al final podremos unir las soluciones separables de tal manera que construyamos la solución

más general.

Para soluciones separables tenemos

(ordinario derivadas, ahora), y la ecuación de Schrödinger dice

O dividiendo por :

(2,3)

Ahora, el lado izquierdo es una función de t solo, y el lado derecho es una función de X solo.2 La única forma en que esto puede

ser posible es si ambas partes están de hecho constante—De lo contrario, variando t, Podría cambiar el lado izquierdo sin tocar

el lado derecho, y los dos ya no serían iguales. (Ese es un argumento sutil pero crucial, así que si es nuevo para usted, asegúrese

de hacer una pausa y pensarlo detenidamente). Por razones que aparecerán en un momento, llamaremos a la separación

constantemi. Luego

(2,4)

43
y

(2,5)

La separación de variables se ha convertido en un parcial ecuación diferencial en dos ordinario ecuaciones

diferenciales (ecuaciones 2.4 y 2.5). El primero de estos es fácil de resolver (simplemente multiplíquelo pordt e integrar);

la solución general es , pero también podríamos absorber la constante C en (ya que la cantidad de

el interés es el producto . Luego3

(2,6)

El segundo (Ecuación 2.5) se llama ecuación de Schrödinger independiente del tiempo; no podemos seguir adelante con él

hasta que se especifique el potencial.

El resto de este capítulo se dedicará a resolver la ecuación de Schrödinger independiente del tiempo, para una variedad de

potenciales simples. Pero antes de llegar a eso, tienes todo el derecho a preguntar:¿Qué tienen de bueno las soluciones separables?

Después de todo, la mayoría soluciones al (tiempo Delawarependiente) Ecuación de Schrödinger hacer no coje la forma

. Ofrezco tres respuestas, dos de ellas físicas y una matemática:


1. Sonestados estacionarios. Aunque la función de onda en sí misma,

(2,7)

depende (obviamente) de t, los densidad de probabilidad,

(2,8)

lo hace no—La dependencia del tiempo se anula.4 Lo mismo sucede al calcular el valor esperado de
cualquier variable dinámica; Ecuación1,36 reduce a

(2,9)

Cada valor esperado es constante en el tiempo; bien podríamos dejar el factor en conjunto, y simplemente

utilizar en lugar de. (De hecho, es común referirse a "la función de onda", pero este es un lenguaje descuidado

que puede ser peligroso, y es importante recordar que elcierto La función de onda siempre lleva ese factor de

ondulación dependiente del tiempo). En particular, es constante, y por lo tanto (Ecuación 1,33)

. Nada jamássucede en un estado estacionario.

2. Son estados deenergía total definida. En mecánica clásica, la energía total (cinética más potencial) se llama
Hamiltoniano:

(2,10)

44
El correspondiente hamiltoniano operador, obtenido por la sustitución canónica por lo , es
tanto5

(2,11)

Así, la ecuación de Schrödinger independiente del tiempo (Ecuación 2.5) puede ser escrito

(2,12)

y el valor esperado de la energía total es

(2,13)

(Nótese que la normalización de implica la normalización de.) Además,

y por lo tanto

Entonces la varianza de H es

(2,14)

Pero recuerda, si , entonces todos los miembros de la muestra deben compartir el mismo valor (la distribución

tiene un margen cero). Conclusión: Una solución separable tiene la propiedad de que Cada medición de la

energía total seguramente devolverá el valor E. (Por eso elegí esa letra para la constante de separación).

3. La solución general es unacombinación lineal de soluciones separables. Como estamos a punto de descubrir, la

ecuación de Schrödinger independiente del tiempo (Ecuación2.5) produce una colección infinita de soluciones

, que escribimos como, cada uno con su constante de separación asociada


; por lo tanto, hay una función de onda diferente para cadaenergía permitida:

Ahora (como puede comprobar fácilmente por sí mismo) el (tiempo-Delawarependiente) Ecuación de Schrödinger (Ecuación 2.1)

tiene la propiedad de que cualquier combinación lineal6 de soluciones es en sí misma una solución. Una vez que hayamos

encontrado las soluciones separables, podemos construir inmediatamente una solución mucho más general, de la forma

(2,15)

Sucede que cada La solución a la ecuación de Schrödinger (dependiente del tiempo) se puede escribir de esta

forma: es simplemente una cuestión de encontrar las constantes correctas para que se ajusten a las condiciones

iniciales del problema en cuestión. Verá en las siguientes secciones cómo funciona todo esto en la práctica, y en

el Capítulo3 lo pondremos en un lenguaje más elegante, pero el punto principal es este: una vez que haya

resuelto el tiempo-enecuación de Schrödinger dependiente, esencialmente hecho; llegar desde allí

45
a la solución general de la época-DelawareLa ecuación de Schrödinger pendiente es, en principio, simple
y directa.

Han pasado muchas cosas en las últimas cuatro páginas, así que permítanme recapitular, desde una perspectiva algo

diferente. Aquí está el problema genérico: se le da un potencial (independiente del tiempo) y la función de onda inicial

; su trabajo es encontrar la función de onda, para cualquier tiempo posteriort. Para hacer esto debes resolver

la ecuación de Schrödinger (dependiente del tiempo) (Ecuación 2.1). La estrategia es primero resolver el tiempo
enecuación de Schrödinger dependiente (Ecuación 2.5); esto produce, en general, un conjunto infinito de
soluciones, cada una con su propia energía asociada,. Para ajustarlo, escriba la combinación lineal general de
estas soluciones:

(2,16)

el milagro es que puedes siempre coincidir con el estado inicial especificado7 mediante la elección adecuada de las constantes Para .
construir, simplemente agregue a cada término su dependencia temporal característica (su "factor de fluctuación"),
:8

(2,17)

Las propias soluciones separables,

(2,18)

están estacionario afirma, en el sentido de que todas las probabilidades y valores de expectativa son independientes del tiempo,

pero esta propiedad es enfáticamente no compartida por la solución general (Ecuación 2.17): las energías son diferentes, para

diferentes estados estacionarios, y las exponenciales no se cancelan, cuando construyes.

Ejemplo 2.1
Suponga que una partícula comienza en una combinación lineal de solo dos estados estacionarios:

(Para simplificar las cosas, asumiré que las constantes y los estados están verdadero.) Cuál es el

función de onda en momentos posteriores? Encuentre la densidad de probabilidad y describa su movimiento.Solución:

La primera parte es sencilla:

dónde y son las energías asociadas con y . Resulta que

La densidad de probabilidad oscila sinusoidalmente, a una frecuencia angular ; este es


ciertamente no un estado estacionario. Pero observe que tomó uncombinación lineal de estados estacionarios (con

46
diferentes energías) para producir movimiento.9

Quizás se esté preguntando cuál será la explicación representar físicamente. Te diré la respuesta, aunque el

de los coeficientes que tendrá que esperar. 3:

(2,19)

Una medición competente siempre dará como resultado uno de los valores "permitidos" (de ahí el nombre), y la es el
probabilidad de obtener el especial valor . 10 Por supuesto, el suma de estas probabilidades debe ser 1:

(2,20)

y el valor esperado de la energía debe ser

(2,21)

Pronto veremos cómo funciona esto en algunos ejemplos concretos. Observe, finalmente, que debido a que las constantes son

independientes del tiempo, también lo es la probabilidad de obtener una energía particular y,con mayor razón, el valor

esperado de H. Estas son manifestaciones deconservación de energía en mecánica cuántica.

∗ Problema 2.1 Demuestre los siguientes tres teoremas:

(a) Para soluciones normalizables, la constante de separación mi debe ser verdadero.

Insinuación:Escribir mi (en ecuación 2,7) como (con y Γ real), y demuestre que si la

Ecuación 1,20 es aguantar para todos t, Γ debe ser cero.

(B) La función de onda independiente del tiempo siempre se puede considerar verdadero(a

diferencia, que es necesariamente complejo). Esto no significa que todas las soluciones a la

ecuación de Schrödinger independiente del tiempoes verdadero; lo que dice es que si tienes

uno que esno, siempre se puede expresar como una combinación lineal de soluciones (con

la misma energía) que están. Vos tambienPodría también apégate a los mensajes que son

reales. Insinuación: Si satisface la ecuación

2.5, para una dada mi, también lo hace su conjugado complejo, y por lo tanto también

las combinaciones lineales reales y .


(C) Si es un incluso función (es decir, siempre se luego pueden

considerará par o impar. Insinuación: Si satisface la ecuación

2.5, para una dada mi, también lo hacen las , y por tanto también los pares e impares

combinaciones lineales .

47
∗ , para
Problema 2.2 Muestra esa mi debe exceder el valor mínimo de solución normalizable a la cada

ecuación de Schrödinger independiente del tiempo. ¿Cuál es el análogo clásico de esta

afirmación?Insinuación: Reescribir la ecuación 2.5 en la forma

si , entonces y su segunda derivada siempre tienen la mismo signo-discutir

que tal función no se puede normalizar.

48
2.2 El pozo cuadrado infinito

Suponer

(2,22)

(Figura 2.1). Una partícula en este potencial es completamente libre, excepto en los dos extremos. y , dónde

una fuerza infinita le impide escapar. Un modelo clásico sería un carro sobre una pista de aire horizontal sin fricción, con parachoques

perfectamente elásticos; simplemente sigue rebotando hacia adelante y hacia atrás para siempre. (Este potencial es artificial, por

supuesto, pero le insto a que lo trate con respeto. A pesar de su simplicidad, o más bien, precisamenteporque de su simplicidad, sirve

como un caso de prueba maravillosamente accesible para toda la maquinaria elegante que viene después. Volveremos a consultarlo con

frecuencia).

Figura 2.1: El potencial de pozo cuadrado infinito (Ecuación 2.22).

Fuera de el pozo, (la probabilidad de encontrar la partícula allí es cero). Dentro el pozo, donde

, la ecuación de Schrödinger independiente del tiempo (Ecuación 2.5) lee

(2,23)

(2,24)

(Al escribirlo de esta manera, he asumido tácitamente que ; lo sabemos por el problema2.2 ese no lo haré

trabajo.) Ecuación 2,24 es el clasico oscilador armónico simple ecuación; la solución general es

(2,25)

dónde A y B son constantes arbitrarias. Normalmente, estas constantes son fijadas por elcondiciones de borde del

problema. Quéestán las condiciones de contorno adecuadas para? Ordinariamente,ambos y son continuos,11 pero

cuando el potencial llega al infinito, sólo se aplica el primero de ellos. (Voy ajustificar estas condiciones de contorno, y

dar cuenta de la excepción cuando, en la Sección 2.5; por ahora espero que confíes en mí.)

La continuidad de requiere que

(2,26)

49
para unirse a la solución fuera del pozo. ¿Qué nos dice esto sobreA y B? Bien,

asi que , y por lo tanto

(2,27)

Luego , entonces tampoco (en cuyo caso nos quedamos con lo trivial, no normalizable,
solución , si no , Lo que significa que

(2,28)

Pero no es bueno (de nuevo, eso implicaría , y las soluciones negativas no aportan nada nuevo,

ya que y podemos absorber el signo menos en A. Entonces eldistinto las soluciones son

(2,29)

Curiosamente, la condición de frontera enk, no determina la constante A, sino más bien la constante
y por tanto los posibles valores de mi:

(2,30)

En contraste radical con el caso clásico, una partícula cuántica en el cuadrado infinito bien no puede tener simplemente alguna energía

vieja: tiene que ser uno de estos valores especiales ("permitidos").12 Encontrar A, nosotros normalizar :13

Esto solo determina el magnitud de A, pero es más sencillo elegir la raíz real positiva: fase de A no (los
tiene ningún significado físico de todos modos). Dentro del pozo, entonces, las soluciones son

(2,31)

Como se prometió, la ecuación de Schrödinger independiente del tiempo ha proporcionado un conjunto infinito de

soluciones (una para cada entero positivo. Las primeras se muestran en la Figura 2.2. Se parecen a las ondas estacionarias en

una cadena de longituda; , que lleva la energía más baja, se llamaestado fundamental, los otros, cuyas energías aumentan en

tener algo
proporción a, se denominan estados emocionados. Como colección, las funciones interesantes e importantes propiedades:

1. Son alternativamente incluso y impar, con respecto al centro del pozo: es par, es impar, es
par, etc.14
2. A medida que aumenta la energía, cada estado sucesivo tiene una nodo (cruce por cero): no tiene ninguno (los

puntos finales no cuentan), tiene uno, tiene dos, etc.

3. Son mutuamente ortogonal, en el sentido de que15

(2,32)

50
Figura 2.2: Los primeros tres estados estacionarios del pozo cuadrado infinito (Ecuación 2,31).

Prueba:

Tenga en cuenta que este argumento no no trabajar si . (¿Puedes identificar el punto en el que falla?) En ese caso

La normalización nos dice que la integral es 1. De hecho, podemos combinar la ortogonalidad y la normalización en una sola

declaración:

(2,33)

dónde (la llamada Delta de Kronecker) es definido por

(2,34)

Decimos que las s son ortonormal.


4. Son completo, en el sentido de que cualquier otro función, , se puede expresar como lineal
combinación de ellos:

(2,35)

No estoy a punto de probar la integridad de las funciones , pero si has estudiado


cálculo avanzado reconocerá que la ecuación 2,35 no es nada más que el series de Fourier porque el hecho de ,y
que "cualquier" función se pueda expandir de esta manera a veces se llama Teorema de dirichlet.dieciséis

Los coeficientes se pueden evaluar, para un dato, mediante un método que llamo El truco de Fourier,
que explota maravillosamente la ortonormalidad de: Multiplica ambos lados de la ecuación 2,35 por e
integrar.

(2,36)

51
(Observe cómo el delta de Kronecker mata todos los términos de la suma excepto aquel para el cual . ) Por lo tanto, la

norteEl coeficiente en la expansión de es17

(2,37)

Estas cuatro propiedades son extremadamente poderosas y no son peculiares del pozo cuadrado infinito. La primera es

cierta siempre que el potencial en sí sea una función simétrica; el segundo es universal, independientemente de la forma del

potencial.18 La ortogonalidad también es bastante general; te mostraré la prueba en el capítulo 3. La integridad es válida para

todos los potenciales que es probable que encuentre, pero las pruebas tienden a ser desagradables y laboriosas; Me temo que

la mayoría de los físicos simplementeasumir integridad y esperanza de lo mejor.

Los estados estacionarios (Ecuación 2.18) del cuadrado infinito bien son

(2,38)

Reclamé (Ecuación 2.17) que la solución más general a la ecuación de Schrödinger (dependiente del tiempo) es una combinación

lineal de estados estacionarios:

(2,39)

(Si duda de que esto es una solución, por supuesto cheque ¡It!) Solo me queda demostrar que puedo ajustar cualquier

función de onda inicial prescrita, mediante la elección adecuada de los coeficientes:

La integridad de la s (confirmada en este caso por el teorema de Dirichlet) garantiza que siempre puedo
expresarme de esta manera, y su ortonormalidad autoriza el uso del truco de Fourier para determinar los
coeficientes reales:

(2,40)

Ese lo hace it: Dada la función de onda inicial, , primero calculamos los coeficientes de expansión,

usando la ecuación 2,40, y luego conéctelos a la ecuación 2,39 para obtener . Armado con la ola
función, estamos en posición de calcular cualquier cantidad dinámica de interés, utilizando los procedimientos
del capítulo 1. Y este mismo ritual se aplica aalguna potencial: las únicas cosas que cambian son la forma
funcional de la sy la ecuación de las energías permitidas.

Ejemplo 2.2
Una partícula en el pozo cuadrado infinito tiene la función de onda inicial

por alguna constante A (ver figura 2.3). Fuera de el pozo, por supuesto, . Encontrar .

52
Figura 2.3: La función de onda inicial en el ejemplo 2.2.

Solución: Primero tenemos que determinar A, al normalizar :

asi que

los norteEl coeficiente es (Ecuación 2,40)

Así (Ecuación 2,39):

Ejemplo 2.3

53
Verifica esa ecuación 2,20 se satisface, para la función de onda en el ejemplo 2.2. Si midió la energía de
una partícula en este estado, ¿cuál es el resultado más probable? ¿Cuál es el valor esperado de la
energía?
Solución: La función de onda inicial (Figura 2.3) se parece mucho al estado fundamental (Figura 2.2).
Esto sugiere que debería dominar,19 y de hecho

El resto de coeficientes componen la diferencia:20

El resultado más probable de una medición de energía es - más del 99,8% de todos
las mediciones darán este valor. El valor esperado de la energía (Ecuación2.21) es

Como era de esperar, está muy cerca de la (5 en lugar de - levemente mas grande, porque

mezcla de estados excitados.

Por supuesto, no es casualidad que la ecuación 2,20 salió bien en el ejemplo 2.3. De hecho, esto se sigue de
la normalización de (los s son independientes del tiempo, así que voy a hacer la prueba; si esto te molesta,
puedes generalizar fácilmente el argumento a arbitrario.

(De nuevo, el delta de Kronecker elige el término en el resumen sobre metro.) Del mismo modo, la expectativa

valor de la energía (Ecuación 2.21) se puede comprobar explícitamente: La ecuación de Schrödinger independiente del tiempo

(Ecuación 2.12) dice

(2,41)

asi que

54
Problema 2.3 Demuestre que no existe una solución aceptable para la ecuación de

Schrödinger (independiente del tiempo) para el pozo cuadrado infinito con o. (Este es un

caso especial del teorema general del problema2.2, pero esta vez hágalo resolviendo

explícitamente la ecuación de Schrödinger y mostrando que no puede satisfacer las

condiciones de contorno).

∗ Problema 2.4 Calcular , y, para el norteth estacionario


estado del pozo cuadrado infinito. Compruebe que se cumple el principio de incertidumbre.

¿Qué estado se acerca más al límite de incertidumbre?

∗ Problema 2.5 Una partícula en el pozo cuadrado infinito tiene como función de onda inicial una mezcla

uniforme de los dos primeros estados estacionarios:

(a) Normalizar . (Es decir, encuentraA. Esto es muy fácil si explota y.


ortonormalidad de Recuerde que, habiendo normalizado en,
puedes estar seguro de que corsé normalizado: si lo duda, compruébelo

explícitamente después de hacer la parte (b)).

(B) Encontrar y . Expresa este último como una función sinusoidal.

de tiempo, como en el ejemplo 2.1. Para simplificar el resultado, deje .


(C) Calcular. Observe que oscila en el tiempo. ¿Cuál es la frecuencia
angular de la oscilación? ¿Cuál es la amplitud de la oscilación? (Si su
amplitud es mayor que, vaya directamente a la cárcel).
(D) Calcular. (Como diría Peter Lorre, "Do it zekveek vay, Johnny! ")
(mi) Si midiera la energía de esta partícula, ¿qué valores podría obtener y
cuál es la probabilidad de obtener cada uno de ellos? Encuentre el valor
esperado deH. ¿Cómo se compara con y?

Problema 2.6 Aunque el en general La constante de fase de la función de onda no


tiene importancia física (se cancela cada vez que calcula una cantidad medible), la
relativo fase de los coeficientes en la ecuación 2.17 lo hace importar. Por ejemplo,
suponga que cambiamos la fase relativa de y en el Problema2.5:

55
donde ϕ es una constante. Encontrar ,y y compare su y.
resultados con lo que obtuviste antes. Estudia los casos especiales

(Para una exploración gráfica de este problema, consulte el subprograma en la nota al pie 9 de este

capítulo.)

∗ Problema 2.7 Una partícula en el pozo cuadrado infinito tiene la función de onda inicial

(a) Bosquejo y determina la constante A.


(B) Encontrar .
(C) ¿Cuál es la probabilidad de que una medición de la energía arroje el
valor?
(D) Encuentre el valor esperado de la energía, usando la ecuación 2.21.21

Problema 2.8 Una partícula de masa metro en el pozo cuadrado infinito (de ancho en que empieza

el estado

por alguna constante A, así es (en igualmente probable que se encuentre en cualquier punto de

la mitad izquierda del pozo. ¿Cuál es la probabilidad de que una medición de la


energía (en algún momento posterior arroje el valor?

Problema 2.9 Para la función de onda en el ejemplo 2.2, encuentre el valor esperado deH,

en un momento, la forma "anticuada":

Compare el resultado que obtuvimos en el Ejemplo 2.3. Nota: Debido es independiente de

al tiempo, no hay pérdida de generalidad en el uso.

56
2.3 El oscilador armónico
El paradigma de un oscilador armónico clásico es una masa metro unido a un resorte de fuerza constante k. El

movimiento se rige porley de Hooke,

(ignorando la fricción), y la solución es

dónde

(2,42)

es la frecuencia (angular) de oscilación. La energía potencial es

(2,43)

su gráfica es una parábola.

Por supuesto, no existe tal cosa como un Perfecto oscilador armónico: si lo estira demasiado, el resorte se romperá

y, por lo general, la ley de Hooke falla mucho antes de que se alcance ese punto. Pero prácticamente cualquier potencial

esaproximadamente parabólico, en la vecindad de un mínimo local (Figura 2.4). Formalmente, si ampliamos

en un Serie de taylor sobre el mínimo:

sustraer (puede agregar una constante a con impunidad, ya que eso no cambia la fuerza),
reconocer que (ya que es un mínimo), y elimine los términos de orden superior (que son insignificantes como

siempre y cuando permanece pequeño), obtenemos

que describe la oscilación armónica simple (sobre el punto , con una constante de resorte efectiva

. Es por eso que el oscilador armónico simple es tan importante: prácticamentealguna el movimiento oscilatorio es

aproximadamente armónico simple, siempre que la amplitud sea pequeña.22

57
Figura 2.4: Aproximación parabólica (curva de trazos) a un potencial arbitrario, en la vecindad de un mínimo
local.

los cuántico El problema es resolver la ecuación de Schrödinger para el potencial

(2,44)

(se acostumbra eliminar la constante de resorte a favor de la frecuencia clásica, usando la Ecuación 2,42). Como hemos

visto, basta con resolver la ecuación de Schrödinger independiente del tiempo:

(2,45)

En la literatura encontrará dos enfoques completamente diferentes de este problema. La primera es una sencilla solución de

"fuerza bruta" para la ecuación diferencial, utilizando lamétodo de serie de potencia; tiene la virtud de que la misma estrategia

se puede aplicar a muchos otros potenciales (de hecho, la usaremos en el capítulo4 para tratar el átomo de hidrógeno). El

segundo es una técnica algebraica diabólicamente inteligente, que utiliza los llamadosoperadores de escalera. Primero te

mostraré el método algebraico, porque es más rápido y simple (y mucho más divertido);23 Si desea omitir el método de la serie

de potencia por ahora, está bien, pero ciertamente debería planear estudiarlo en algún momento.

58
2.3.1 Método algebraico

Para empezar, reescribamos la ecuación 2,45 en una forma más sugerente:

(2,46)

dónde es el operador de impulso.24 La idea básica es factor el hamiltoniano,

(2,47)

Si estos fueran números, sería fácil:

Aquí, sin embargo, no es tan simple, porqueviajar y X están operadores, y los operadores, en general,
diariamente no es lo mismo que, como veremos en un momento, aunque es posible que desee detenerse ahora y

piénselo bien). Sin embargo, esto nos motiva a examinar las cantidades

(2,48)

(el factor al frente está ahí para que los resultados finales se vean más agradables).

Bien que es el producto ?

Como se anticipó, hay un término adicional, que involucra . A esto lo llamamos elconmutador de X y ; es un
medida de lo mal que fallar para viajar. En general, el conmutador de operadores entre y (escrito con
corchetes) es

(2,49)

En esta notación,

(2,50)

Necesitamos descubrir el conmutador de X y . Advertencia: Los operadores son notoriamente resbaladizos


para trabajar en abstracto, y es probable que cometa errores a menos que les dé una "función de prueba" para
actuar. Al final, puede desechar la función de prueba y se quedará con una ecuación que involucra solo a los
operadores. En el presente caso tenemos:

(2,51)

59
Descartando la función de prueba, que ha cumplido su propósito,

(2,52)

Esta hermosa y omnipresente fórmula se conoce como la relación de conmutación canónica.25

Con esto, Ecuación 2,50 se convierte en

(2,53)

(2,54)

Evidentemente, el hamiltoniano no factor a la perfección, existe ese extra A la derecha. Note que el
ordenar de y es importante aquí; el mismo argumento, con a la izquierda, produce

(2,55)

En particular,

(2,56)

Mientras tanto, el hamiltoniano se puede escribir igualmente bien

(2,57)

En términos de , entonces, la ecuación de Schrödinger26 porque el oscilador armónico toma la forma

(2,58)

(en ecuaciones como esta, lee los signos superiores a lo ancho, o bien los signos inferiores).

Ahora, aquí viene el paso crucial: afirmo que:

Si satisface la ecuación de Schrödinger con energía mi (es decir: , luego satisface el


Ecuación de Schrödinger con energía : .

Prueba:

60
(Usé la ecuación 2,56 para por en la segunda línea. Tenga en cuenta que mientras

reemplazar el pedido deloy hace asunto, el orden de que el y cualquierconstantes—Como, y mi-lo hace no;

un operador conmuta con cualquier constante).

Del mismo modo, es una solución con energía. :

Aquí, entonces, hay una máquina maravillosa para generar nuevas soluciones, con energías más altas y más bajas, si

pudiéramos encontrar uno solución, para empezar! Invocamos operadores


en energía; esde escalera,de
eloperador porque nos permiten
elevación, subir
y en la figura y el
2.5.

operador de bajada. Se ilustra la "escalera" de los estados

Figura 2.5: La "escalera" de estados para el oscilador armónico.

¡Pero espera! ¿Qué pasa si aplico el operador de descenso repetidamente? Eventualmente voy a alcanzar un estado

con energía menor que cero, que (de acuerdo con el teorema general del Problema2.3) ¡no existe! En algún momento la

máquina debe fallar. ¿Cómo puede pasar eso? Sabemos que es una nueva solución para Schrödinger

61
ecuación, pero no hay garantía de que sea normalizable—Podría ser cero, o su integral cuadrada podría ser infinita. En

la práctica es lo primero: se produce un "peldaño más bajo" (llámelo de modo que

(2,59)

Podemos usar esto para determinar :

Esta ecuación diferencial es fácil de resolver:

asi que

Bien podríamos normalizarlo de inmediato:

asi que , y por lo tanto

(2,60)

Para determinar la energía de este estado, la introducimos en la ecuación de Schrödinger (en forma de Ecuación 2,58),

y explotar el hecho de que:

(2,61)

Con nuestro pie ahora firmemente plantado en el peldaño inferior (el estado fundamental del oscilador cuántico),

simplemente aplicamos el operador de elevación (repetidamente) para generar los estados excitados,27 aumentando la energía

con cada paso:

(2,62)

dónde es la constante de normalización. Aplicando el operador de elevación (repetidamente) a , entonces, podemos (en

principio) construir todo28 los estados estacionarios del oscilador armónico. Mientras tanto, sin hacer eso

explícitamente, ¡hemos determinado las energías permitidas!

62
Ejemplo 2.4
Encuentre el primer estado excitado del oscilador armónico.

Solución: Usando la ecuación 2,62,

(2,63)

Podemos normalizarlo "a mano":

así que, como sucede, .


No quisiera calcular de esta manera (¡aplicando el operador de elevación cincuenta veces!), pero nunca

mente: en principio Ecuación 2,62 hace el trabajo, excepto por la normalización.

Incluso puede obtener la normalización algebraicamente, pero requiere un juego de pies elegante, así que observe de

cerca. Sabemos que esproporcional para ,

(2,64)

pero cuales son los factores de proporcionalidad, y ? Primero tenga en cuenta que para "cualquiera"29 funciones y ,

(2,65)

En el lenguaje del álgebra lineal, es el conjugado ermitaño (o adjunto) de .

Prueba:

y la integración por partes toma para (los términos de la frontera desaparecen, porque

la razón indicada en la nota a pie de página 29), asi que

En particular,

Pero (invocando ecuaciones 2,58 y 2,62)

63
(2,66)

asi que

Pero desde y están normalizados, se sigue que y , y por lo tanto30

(2,67)

Por lo tanto

etcétera. Claramente

(2,68)

lo que quiere decir que el factor de normalización en la ecuación 2,62 está confirmando (en particular, ,
nuestro resultado en el ejemplo 2.4).

Como en el caso del pozo cuadrado infinito, los estados estacionarios del oscilador armónico son ortogonales:

(2,69)

Esto se puede demostrar usando la ecuación 2,66y ecuación 2,65 dos veces, primero en movimiento y luego moviéndose :

A no ser que , luego, debe ser cero. La ortonormalidad significa que podemos volver a utilizar el truco de Fourier

(Ecuación 2,37) para evaluar los coeficientes, cuando expandimos como una combinación lineal de estacionario

estados (Ecuación 2.16). Como siempre, es la probabilidad de que una medición de la energía arroje el
valor.

64
Ejemplo 2.5
Encuentre el valor esperado de la energía potencial en el nortea estado estacionario del oscilador
armónico.
Solución:

Hay un hermoso dispositivo para evaluar integrales de este tipo (que involucra poderes de X o : Utilizar el

definición (Ecuación 2,48) para expresar X y en cuanto a los operadores de subida y bajada:

(2,70)

En este ejemplo estamos interesados en :

Entonces

Pero es (aparte de la normalización) , que es ortogonal a, y lo mismo ocurre con


, que es proporcional a. Entonces esos términos se eliminan y podemos usar la ecuación2,66 para

evaluar los dos restantes:

Da la casualidad de que el valor esperado de la energía potencial es exactamente mitad el total (la otra mitad, por

supuesto, es cinética). Esta es una peculiaridad del oscilador armónico, como veremos más adelante (Problema3.37).

∗ Problema 2.10

(a) Construir .
(B) Bosquejo ,y .
(C) Compruebe la ortogonalidad de ,y , por integración explícita. Insinuación:

Si explota la uniformidad y la imparcialidad de las funciones, en realidad solo

queda una integral por hacer.

∗ Problema 2.11

(a) Calcular ,y , para los estados (Ecuación 2,60) y


(Ecuación 2,63), por integración explícita. Comentario: En este y otros
problemas relacionados con el oscilador armónico, simplifica las cosas si

sesenta y cinco
introducir los variable y los constante

.
(B) Compruebe el principio de incertidumbre para estos estados.

(C) Calcule y para estos estados. (¡No se permiten nuevas integraciones!) ¿Es su
suma lo que cabría esperar?

∗ Problema 2.12 Encontrar , y, para el norteth estado estacionario de

el oscilador armónico, usando el método del Ejemplo 2.5. Compruebe que se cumple
el principio de incertidumbre.

Problema 2.13 Una partícula en el potencial del oscilador armónico comienza en el estado

(a) Encontrar A.

(B) Construir y . No se emocione demasiado si

oscila exactamente a la frecuencia clásica; que hubiera sido si hubiera


especificado , en lugar de ?31
(C) Encontrar y . Compruebe que el teorema de Ehrenfest (Ecuación1,38) sostiene,

para esta función de onda.

(D) Si midiera la energía de esta partícula, ¿qué valores podría obtener y con
qué probabilidades?

66
2.3.2 Método analítico

Regresemos ahora a la ecuación de Schrödinger para el oscilador armónico,

(2,71)

y resolverlo directamente, por el método de la serie de potencias. Las cosas se ven un poco más limpias si introducimos la variable

adimensional

(2,72)

en términos de ξ la ecuación de Schrödinger dice

(2,73)

dónde K es la energía, en unidades de :

(2,74)

Nuestro problema es resolver la ecuación 2,73, y en el proceso obtener los valores "permitidos" de K (y por lo tanto de .
Para empezar, tenga en cuenta que en muy grande ξ (es decir, en muy grande domina completamente sobre la constante

K, entonces en este régimen

(2,75)

que tiene la solución aproximada (¡compruébalo!)

(2,76)

los B El término claramente no es normalizable (explota al igual ; las soluciones físicamente aceptables, entonces,

que la forma asintótica

(2,77)

Esto sugiere que "pelemos" la parte exponencial,

(2,78)

con la esperanza de que lo que queda, , tiene una forma funcional más simple que sí mismo.32 Ecuación de diferenciación

2,78,

67
entonces la ecuación de Schrödinger (Ecuación 2,73) se convierte en

(2,79)

Propongo buscar soluciones a la ecuación 2,79 en forma de serie de potencia en ξ:33

(2,80)

Diferenciando la serie término por término,

Poner estos en ecuación 2,80, encontramos

(2,81)

Se sigue (de la singularidad de las expansiones de series de potencias34 ) que el coeficiente de cada poder de ξ debe

desaparecer,

y de ahí que

(2,82)

Esta fórmula de recursividad es completamente equivalente a la ecuación de Schrödinger. A partir de,


genera todos los coeficientes pares:

y a partir de, genera los coeficientes impares:

Escribimos la solución completa como

(2,83)

68
dónde

es una función par de ξ, construida sobre y

es una función extraña, basada en. Así ecuación2,82 determina en términos de dos constantes arbitrarias

y - que es justo lo que esperaríamos, para una ecuación diferencial de segundo orden.
Sin embargo, no todas las soluciones así obtenidas son normalizable. Para en muy grandej, la fórmula de recursividad se convierte

en (aproximadamente)

con la solución (aproximada)

por alguna constante C, y esto produce (en general ξ, donde dominan los poderes superiores)

Ahora si h va como , luego (¿recuerdas? Eso es lo que estamos tratando de calcular) dice
(Ecuación 2,78), que es precisamente el comportamiento asintótico que no lo hice querer.35 Sólo hay uno

manera de salirse de esto: para soluciones normalizables la serie de potencia debe terminar. Debe ocurrir algo "más

alto"j (llámalo , de manera que la fórmula de recursividad escupe; (esto truncará cualquiera la serie si
o las series losotro uno debe ser cero desde el principio: norte es par, y si norte es impar).
Para soluciones físicamente aceptables, entonces, Ecuación 2,82 requiere que

para algún entero positivo norte, es decir (refiriéndose a la ecuación 2,74) que el energía debe ser

(2,84)

Así recuperamos, por un método completamente diferente, la condición de cuantificación fundamental que encontramos

algebraicamente en la Ecuación 2,62.

Al principio parece bastante sorprendente que la cuantificación de la energía surja de un detalle técnico en la solución de

la serie de potencias de la ecuación de Schrödinger, pero veámoslo desde una perspectiva diferente. Ecuación2,71 tiene

soluciones, por supuesto, para alguna valor de mi (de hecho, tiene dos soluciones linealmente independientes para todos. Pero

casi todas estas soluciones explotan exponencialmente en general.X, y por lo tanto no son normalizables. Imagine, por ejemplo,

utilizar unmi eso es un poco menos que uno de los valores permitidos (digamos,, y trazando la solución: Figura 2.6(a). Ahora

prueba unmi levemente mas grande (decir, ; la "cola" ahora explota en elotro dirección (Figura 2.6(B)). A medida que modifica el

parámetro en pequeños incrementos de 0,49 a 0,51, el gráfico

69
“Da la vuelta” precisamente en el valor 0,5; sólo aquí la solución escapa al crecimiento asintótico exponencial que
la vuelve físicamente inaceptable.36

Figura 2.6: Soluciones de la ecuación de Schrödinger para (a) , y B) .

Para los valores permitidos de K, la fórmula de recursión dice

(2,85)

Si , solo hay un término en la serie (debemos elegir: matar ,y en ecuación 2,85


rendimientos

y por lo tanto

(que, además de la normalización, reproduce la Ecuación 2,60). Para nosotros tomamos ,37 y ecuación
2,85 con rendimientos, entonces

y por lo tanto

70
(confirmando la ecuación 2,63). Para rendimientos ,y da , asi que

etcétera. (Compare el problema2.10, donde este último resultado se obtuvo por medios algebraicos).

En general, será un polinomio de grado norte en ξ, involucrando poderes pares solamente, si norte es un número

entero par, y las potencias impares solo, si norte es un número entero impar. Aparte del factor global o son los llamados

Polinomios de Hermite,. 38 Los primeros se enumeran en la tabla 2.1. Por tradición, el factor

multiplicativo arbitrario se elige de modo que el coeficiente de la potencia más alta de ξ sea. Con esta convención, el

normalizado39 Los estados estacionarios para el oscilador armónico son

(2,86)

Son idénticos (por supuesto) a los que obtuvimos algebraicamente en la Ecuación 2,68.

Cuadro 2.1: Los primeros polinomios de Hermite, .

En figura 2,7(a) he trazado para los primeros nortes. El oscilador cuántico es sorprendentemente diferente

de su contraparte clásica, no sólo se cuantifican las energías, sino que las distribuciones de posición tienen algunas

características extrañas. Por ejemplo, la probabilidad de encontrar la partícula fuera del rango permitido clásicamente

(es decir, conX mayor que la amplitud clásica para la energía en cuestión) es no cero (ver Problema 2.14), y en todos los

estados impares la probabilidad de encontrar la partícula en el centro es cero. Solo en generalnorte empezamos a ver

alguna semejanza con el caso clásico. En figura2,7(b) He superpuesto la distribución de posición clásica (Problema 1,11)

en el cuántico (porque; si suavizaras las protuberancias, las dos encajarían bastante bien.

71
Figura 2.7: (a) Los primeros cuatro estados estacionarios del oscilador armónico. (b) Gráfico de , con el
distribución clásica (curva discontinua) superpuesta.

Problema 2.14 En el estado fundamental del oscilador armónico, ¿cuál es la probabilidad

(correcta a tres dígitos significativos) de encontrar la partícula fuera de la región

clásicamente permitida? Insinuación: Clásicamente, la energía de un oscilador es

, dónde a es la amplitud. Así que el "clásico


región permitida "para un oscilador de energía mi se extiende desde hasta
. Busque en una tabla matemática en "Distribución normal" o "Error
Función ”para el valor numérico de la integral, o evaluarlo por computadora.

72
Problema 2.15 Utilice la fórmula de recursividad (Ecuación 2,85) hacer ejercicio y
. Invoque la convención de que el coeficiente de la potencia más alta de ξ es

para fijar la constante general.

∗∗ Problema 2.16 En este problema exploramos algunos de los teoremas más útiles

(enunciados sin prueba) que involucran polinomios de Hermite.

(a) los Fórmula de Rodrigues dice que

(2,87)

Úselo para derivar y .


(B) La siguiente relación de recursión le da en términos de los dos

polinomios de Hermite anteriores:

(2,88)

Úselo, junto con su respuesta en (a), para obtener y .


(C) Si diferencia un nortepolinomio de orden th, obtienes un polinomio de
orden. Para los polinomios de Hermite, de hecho,

(2,89)

Compruebe esto, diferenciando y .


(D) es el norteth z-derivado, en , de El función generadora
; o, para decirlo de otra manera, es el coeficiente de

en la expansión de la serie Taylor para esta función:

(2,90)

Utilice esto para obtener ,y.

73
2.4 La partícula libre
Pasamos al lado de lo que deberían ha sido el caso más simple de todos: la partícula libre En todas partes).

Clásicamente, esto sería solo un movimiento a velocidad constante, pero en la mecánica cuántica el problema es

sorprendentemente sutil. La ecuación de Schrödinger independiente del tiempo dice

(2,91)

(2,92)

Hasta ahora, es lo mismo que dentro del pozo cuadrado infinito (Ecuación 2,24), donde el potencial también es cero; esta vez, sin

embargo, prefiero escribir la solución general en forma exponencial (en lugar de senos y cosenos), por razones que aparecerán

a su debido tiempo:

(2,93)

A diferencia del pozo cuadrado infinito, no hay condiciones de contorno para restringir los posibles valores de k (y por

tanto de; la partícula libre puede llevaralguna (energía positiva. Añadiendo la dependencia del tiempo estándar,

(2,94)

Ahora, alguna funcion de X y t que depende de estas variables en la combinación especial (por
alguna constante representa una onda de forma inmutable, viajando en la dirección - a velocidad v: Un punto fijo
en la forma de onda (por ejemplo, un máximo o un mínimo) corresponde a un valor fijo del argumento y, por lo
tanto, a X y t tal que

Dado que cada punto de la forma de onda se mueve con la misma velocidad, su forma no cambia a medida que se propaga. Por

tanto, el primer término de la ecuación2,94 representa una ola que viaja al Derecha, y el segundo representa una onda (de la

misma energía) que va al izquierda. Por cierto, ya que solo se diferencian por elfirmar en frente de k, bien podríamos escribir

(2,95)

y deja k ejecutar negativo para cubrir el caso de ondas que viajan hacia la izquierda:

(2,96)

Evidentemente, los "estados estacionarios" de la partícula libre son ondas de propagación; su longitud ,
de onda es y, de acuerdo con la fórmula de De Broglie (Ecuación1,39), llevan impulso

74
(2,97)

La velocidad de estas ondas (el coeficiente de t sobre el coeficiente de es

(2,98)

Por otro lado, el clásico velocidad de una partícula libre con energía mi está dada por (puro

cinética, ya que, entonces

(2,99)

Aparentemente, la función de onda de la mecánica cuántica viaja a mitad la velocidad de la partícula que se supone que

representa. Regresaremos a esta paradoja en un momento; hay un problema aún más serio que debemos enfrentar

primero:Esta función de onda no es normalizable:

(2.100)

En el caso de la partícula libre, entonces, las soluciones separables no representan estados físicamente realizables. Una partícula libre no

puede existir en un estado estacionario; o, para decirlo de otra manera,no existe tal cosa como una partícula libre con una energía

definida.

Pero eso no significa que las soluciones separables no nos sirvan de nada. Porque juegan unmatemático papel que es

completamente independiente de su físico interpretación: La solución general de la ecuación de Schrödinger dependiente del

tiempo sigue siendo una combinación lineal de soluciones separables (solo que esta vez es una integral sobre el continuo

variable k, en lugar de un suma sobre el discreto índice:

(2.101)

(La cantidad se factoriza por conveniencia; que juega el papel del coeficiente en ecuación

2.17 es la combinación . ) Ahoraesta función de onda pueden ser normalizado (para


. Pero necesariamente lleva undistancia de ks, y por tanto una gama de energías y velocidades. Lo llamamos unola

paquete.40

En el problema cuántico genérico, estamos dado , y se nos pide que encontrar . Gratis
partícula la solución toma la forma de Ecuación 2.101; la única pregunta es cómo determinar la coincidencia de para
la función de onda inicial:

(2.102)

Este es un problema clásico en el análisis de Fourier; la respuesta es proporcionada porTeorema de plancherel (ver problema

2.19):

(2.103)

75
se llama el Transformada de Fourier de ; es el transformada inversa de Fourier de (el único
la diferencia es el signo del exponente).41 Por supuesto, hay algunas restricciones sobre las funciones permitidas: las

integrales deben existe.42 Para nuestros fines, esto está garantizado por el requisito físico de que se normalice.

Entonces, la solución al problema cuántico genérico, para la partícula libre, es la ecuación2.101, con

(2.104)

Ejemplo 2.6
Una partícula libre, que inicialmente se localiza en el rango , se lanza a la hora :

dónde A y a son constantes reales positivas. Encontrar .


Solución: Primero tenemos que normalizar :

A continuación calculamos , usando la ecuación 2.104:

Finalmente, volvemos a conectar esto a la ecuación 2.101:

(2.105)

Desafortunadamente, esta integral no se puede resolver en términos de funciones elementales, aunque, por supuesto,

se puede evaluar numéricamente (Figura 2.8). (De hecho, hay muy pocos casos en los que la integral para

(Ecuación 2.101) pueden realizarse explícitamente; ver problema2.21 para una particularmente hermosa

ejemplo.)

76
Figura 2.8: Gráfico de (Ecuación 2.105) a (el rectángulo) y en (los
curva).

En figura 2.9 Yo he trazado y . Tenga en cuenta que para pequeñosa, es estrecho (en, mientras

es amplio (en, y viceversa para grandes a. Perok está relacionado con el impulso, por la ecuación 2,97, por lo que

esta es una manifestación del principio de incertidumbre: la posición puede estar bien definida (pequeña, o el impulso

(grande, pero no ambos.

Figura 2.9: (a) Gráfica de . (b) Gráfico de .

Vuelvo ahora a la paradoja señalada anteriormente: el hecho de que la solución separable viaja en el
Velocidad "incorrecta" para la partícula que aparentemente representa. Estrictamente hablando, el problema se evaporó cuando

descubrimos que no es un estado físicamente realizable. Sin embargo, es interesante averiguar cómo la información sobre la

velocidad de la partículaes contenida en la función de onda (Ecuación 2.101). La idea esencial es la siguiente: un paquete de

ondas es una superposición de funciones sinusoidales cuya amplitud está modulada por ϕ (Figura2.10); consta de "ondas"

contenidas dentro de un "sobre". Lo que corresponde a la velocidad de las partículas no es la velocidad de las ondas individuales

(las llamadasvelocidad de fase), sino más bien la velocidad de la envolvente (la velocidad de grupo) —Que, dependiendo de la

naturaleza de las ondas, puede ser mayor, menor o igual que la velocidad de las ondas que la componen. Para ondas en una

cuerda, la velocidad de grupo es la misma que la velocidad de fase. Para las ondas de agua, es la mitad de la velocidad de fase,

como puede haber notado cuando arroja una piedra a un estanque (si se concentra en una onda en particular, verá que se

acumula desde la parte trasera, avanza a través del grupo, y se desvanecen en el frente, mientras que el grupo en su conjunto

se propaga a la mitad de esa velocidad). Lo que necesito mostrar es que para la función de onda de una partícula libre en

mecánica cuántica, la velocidad del grupo esdos veces la velocidad de fase, justo para igualar la velocidad clásica de las

partículas.

77
Figura 2.10: Un paquete de ondas. La "envolvente" viaja a la velocidad del grupo; las "ondas" viajan a la velocidad de fase.

El problema, entonces, es determinar la velocidad de grupo de un paquete de ondas con la forma genérica

(2.106)

En nuestro caso , pero lo que tengo que decir ahora se aplica a alguna tipo de paquete de ondas, independientemente de su

relación de dispersión (la fórmula para ω en función de. Asumamos que tiene un pico estrecho sobre

algún valor particular. (No hay nadailegal sobre una amplia difusión en k, pero tales paquetes de ondas cambian de forma

rápidamente: diferentes componentes viajan a diferentes velocidades, por lo que toda la noción de un "grupo", con una

velocidad bien definida, pierde su significado.) Dado que el integrando es insignificante excepto en las proximidades de,

podemos como Bueno, Taylor-expanda la función sobre ese punto y mantenga solo los términos principales:

dónde es la derivada de ω con respecto a k, en el punto .


Cambiar variables de k para (centrar la integral en , tenemos

(2.107)

El término en frente es una onda sinusoidal (las "ondas"), viajando a velocidad . Está modulado por la integral
(la "envolvente"), que es una función de y, por lo tanto, se propaga a la velocidad. Por lo tanto, lafasela velocidad
es

(2.108)

mientras que la grupo la velocidad es

(2.109)

(ambos evaluados en .
En nuestro caso, , asi que , mientras que , que es dos veces mejor.

Esto confirma que la velocidad de grupo del paquete de ondas coincide con la velocidad de partícula clásica:

(2.110)

∗ Problema 2.17 Muestra esa y están

formas equivalentes de escribir la misma función de Xy determinar las constantes Cy


D en términos de A y B, y viceversa. Comentario: En mecánica cuántica, cuando las
exponenciales representan de viaje ondas, y son más convenientes para discutir la
partícula libre, mientras que los senos y cosenos corresponden a de pieondas, que
surgen naturalmente en el caso del pozo cuadrado infinito.

78
Problema 2.18 Encuentra la corriente de probabilidad, J (Problema 1,14) para la ecuación de la

función de onda de partícula libre 2,95. ¿En qué dirección fluye la probabilidad?

∗∗ Problema 2.19 Este problema está diseñado para guiarlo a través de una "prueba" del teorema

de Plancherel, comenzando con la teoría de la serie ordinaria de Fourier en unfinito intervalo, y

permitir que ese intervalo se expanda hasta el infinito.

(a) El teorema de Dirichlet dice que "cualquier" función en el intervalo


se puede ampliar como una serie de Fourier:

Demuestre que esto se puede escribir de manera equivalente como

¿Qué es, en términos de y?


(B) Demuestre (mediante la modificación apropiada del truco de Fourier) que

(C) Eliminar norte y a favor de las nuevas variables y


. Muestre que (a) y (b) ahora se convierten

dónde es el incremento en k de uno norte a la siguiente.

(D) Toma el limite para obtener el teorema de Plancherel. Comentario: En vista

de sus orígenes bastante diferentes, es sorprendente (y delicioso) que las


dos fórmulas, una para en términos de, la otra para términos de, tengan en
una estructura tan similar en el límite .

Problema 2.20 Una partícula libre tiene la función de onda inicial.

dónde A y a son constantes reales positivas.

(a) Normalizar .
(B) Encontrar .
(C) Construir , en forma de integral.
(D) Discutir los casos limitantes muy grande, y a muy pequeña).

79
∗ Problema 2.21 El paquete de ondas gaussianas. Una partícula libre tiene la función de onda

inicial.

dónde A y a son constantes (reales y positivas).


(a) Normalizar .
(B) Encontrar . Insinuación: Integrales de la forma

se puede manejar “completando el ,


cuadrado”: Let y note eso. Respuesta:

(2.111)
(C) Encontrar . Exprese su respuesta en términos de la cantidad

Bosquejo (como una función de a , y de nuevo para algunos muy grandes t.

Cualitativamente, ¿qué sucede con , ¿Conforme pasé él tiempo?

(D) Encontrar ,y. Respuesta parcial: , pero

puede ser necesario un poco de álgebra para reducirlo a esta forma simple.

(mi) ¿Se mantiene el principio de incertidumbre? A qué horat ¿El sistema se acerca

más al límite de incertidumbre?

80
2.5 El potencial de la función delta

81
2.5.1 Estados limitados y estados de dispersión

Hemos encontrado dos tipos muy diferentes de soluciones para la ecuación de Schrödinger independiente del tiempo:

para el pozo cuadrado infinito y el oscilador armónico son normalizabley etiquetado con un índice discreto n; por la

partícula libre que sonno normalizabley etiquetado con un variable continua k. Los primeros representan estados

físicamente realizables por derecho propio, los segundos no; pero en ambos casos la solución general a la ecuación de

Schrödinger dependiente del tiempo es una combinación lineal de estados estacionarios; para el primer tipo, esta

combinación toma la forma de unsuma (más, mientras que para el segundo es un integral (sobre . ¿Cuál es el significado

físico de esta distinción?

En clásico mecánica un potencial unidimensional independiente del tiempo puede dar lugar a dos tipos de

movimiento bastante diferentes. Si se eleva más que la energía total de la partícula en cualquier lado (Figura2.11(a)),

entonces la partícula está "atascada" en el pozo potencial: se balancea hacia adelante y hacia atrás entre los puntos de

inflexión, pero no puede escapar (a menos que, por supuesto, le proporcione una fuente de energía extra, como un

motor, pero no estamos hablando de eso). A esto lo llamamos unestado obligado. Si, por otro lado,mi excede en un

lado (o ambos), entonces la partícula viene del "infinito", se ralentiza o acelera bajo la influencia del potencial y vuelve al

infinito (Figura 2.11(B)). (No puede quedar atrapado en el potencial a menos que exista algún mecanismo, como la

fricción, paradisipar energía, pero de nuevo, no estamos hablando de eso). estado de dispersión. Algunos potenciales

solo admiten estados ligados (por ejemplo, el oscilador armónico); algunos solo permiten estados de dispersión (una

colina potencial sin caídas, por ejemplo); algunos permiten ambos tipos, dependiendo de la energía de la partícula.

82
Figura 2.11: (a) Un estado ligado. (b) Estados de dispersión. (c) Aclásico estado ligado, pero un estado de dispersión cuántica.

Los dos tipos de soluciones de la ecuación de Schrödinger corresponden precisamente a los estados límite y de dispersión.

La distinción es aún más clara en el dominio cuántico, porque el fenómeno detunelización (a lo que llegaremos en breve)

permite que la partícula "se filtre" a través de cualquier barrera de potencial finito, por lo que lo único que importa es el

potencial en el infinito (Figura 2.11(C)):

(2.112)

En la vida real, la mayoría de los potenciales van a cero en el infinito, en cuyo caso el criterio se simplifica aún más:

(2.113)

Debido a que el pozo cuadrado infinito y los potenciales del oscilador armónico van al infinito como , admiten
estados vinculados solamente; debido a que el potencial de partículas libres es cero en todas partes, solo permite estados de dispersión.43 En

83
Esta sección (y la siguiente) exploraremos los potenciales que apoyan ambos tipos de estados.

84
2.5.2 El pozo de función delta

los Función delta de Dirac es un pico infinitamente alto, infinitesimalmente estrecho en el origen, cuyo zona es 1

(Figura 2.12):

(2.114)

Técnicamente, no es una función en absoluto, ya que no es finita en (los matemáticos lo llaman un generalizado

función, o distribución).44 Sin embargo, es una construcción extremadamente útil en física teórica. (Por
ejemplo, en electrodinámica la cargadensidad de una carga puntual es una función delta.) Observe que
sería un pico del área 1 en el punto a. Si multiplica por unordinario función, es lo mismo que multiplicar
por,

(2.115)

porque el producto es cero de todos modos excepto en el punto a. En particular,

(2.116)

Esa es la propiedad más importante de la función delta: bajo el signo integral, sirve para "seleccionar" el
valor de en el punto a. (Por supuesto, la integral no tiene por qué ir del dominio de integración incluye
; todo lo que importa es el
que

puntoa, por lo que haría, para cualquier .)

Figura 2.12: La función delta de Dirac (Ecuación 2.114).

Consideremos un potencial de la forma

(2.117)

donde α es una constante positiva.45 Este es un potencial artificial, sin duda (también lo era el cuadrado infinito),
pero es deliciosamente simple de trabajar e ilumina la teoría básica con un mínimo de desorden analítico. La
ecuación de Schrödinger para la función delta lee bien

(2.118)

produce ambos estados ligados y estados de dispersión .


Primero veremos los estados ligados. En la región , asi que

85
(2.119)

dónde

(2.120)

es negativo, por supuesto, por lo que κ es real y positivo.) La solución general de la ecuación 2.119 es

(2.121)

pero el primer término explota como , entonces debemos elegir :

(2.122)

En la región es de nuevo cero, y la solución general es de la forma

; esta vez es el segundo término que explota (como , asi que

(2.123)

Solo queda unir estas dos funciones, usando las condiciones de contorno apropiadas en
. Cité anteriormente las condiciones de contorno estándar para:

(2.124)

En este caso, la primera condición de frontera nos dice que , asi que

(2,125)

se traza en la Figura 2.13. La segunda condición de frontera no nos dice nada; este es (como las paredes
del pozo cuadrado infinito) el caso excepcional dondeV es infinito en la unión, y está claro en el gráfico que esta
función tiene una torcedura. Además, hasta este punto, la función delta no ha entrado en la historia en absoluto.
Resulta que la función delta determina eldiscontinuidad en la derivada de en . Le mostraré ahora cómo funciona
esto y, como subproducto, veremos por qué normalmente es continuo.

Figura 2.13: Función de onda de estado limitado para el potencial de función delta (Ecuación 2.125).

La idea es integrar la ecuación de Schrödinger, de para , y luego tomar el límite como :

86
(2.126)

La primera integral no es más que , evaluado en los dos puntos finales; la última integral escero, en el limite

, ya que es el área de una astilla con ancho de fuga y altura finita. Por lo tanto

(2.127)

Por lo general, el límite de la derecha vuelve a ser cero, y es por eso que es normalmente continuo. Pero cuando

es infinito en el límite, este argumento falla. En particular, si , Ecuación 2.116 rendimientos

(2.128)

Para el caso que nos ocupa (Ecuación 2.125),

y por lo tanto .Y . Entonces ecuación2.128 dice

(2.129)

y la energía permitida (Ecuación 2.120) es

(2.130)

Finalmente, normalizamos:

entonces (eligiendo la raíz real positiva):

(2.131)

Evidentemente, el delta funciona bien, independientemente de su "fuerza" α, tiene exactamente uno estado vinculado:

(2.132)

Qué pasa dispersión estados, con ? Para la ecuación de Schrödinger dice

dónde

87
(2.133)

es real y positivo. La solución general es

(2.134)

y esta vez no podemos descartar ninguno de los dos términos, ya que ninguno de los dos explota. Del mismo modo, para ,

(2.135)

La continuidad de a requiere que

(2.136)

Los derivados son

y por lo tanto . Mientras tanto, , entonces el segundo límite

condición (Ecuación 2.128) dice

(2.137)

o, de forma más compacta,

(2.138)

Habiendo impuesto ambas condiciones de contorno, nos quedamos con dos ecuaciones (Ecuaciones 2.136 y 2.138) en

cuatro incógnitas, B, F, y - cinco, si cuentas k. La normalización no ayudará, este no es un estado


normalizable. Quizás será mejor que hagamos una pausa, entonces, y examinemos el significado físico de estas diversas

constantes. Recuerde que da lugar (cuando se combina con el factor de ondulación a una función de onda que se

propaga alDerecha, y conduce a una onda que se propaga al izquierda. Resulta queA (en ecuación 2.134) es la amplitud

de una onda que llega desde la izquierda, B es la amplitud de una onda que regresa hacia la izquierda; F (Ecuación 2.135

) es la amplitud de una onda que se desplaza hacia la derecha, y GRAMO es la amplitud de una onda que viene desde la

derecha (ver Figura 2.14). En un experimento de dispersión típico, las partículas se disparan desde una dirección,

digamos, desde la izquierda. En ese caso, la amplitud de la onda procedente delDerecha estaráncero:

(2.139)

A es la amplitud de la ola incidente, B es la amplitud de la onda reflejada, y F es la amplitud de la onda


transmitida. Resolver ecuaciones2.136 y 2.138 por B y F, encontramos

(2.140)

(Si desea estudiar la dispersión desde el Derecha, ; luegoGRAMO es la amplitud incidente, F es el reflejado

establezca la amplitud y B es la amplitud transmitida.)

88
Figura 2.14: Dispersión de una función delta bien.

Ahora, la probabilidad de encontrar la partícula en una ubicación específica viene dada , entonces el relativo46

por la probabilidad de que una partícula incidente se refleje es

(2.141)

R se llama el coeficiente de reflexión. (Si tienes unHaz de partículas, te dice el fracción del número
entrante que se recuperará.) Mientras tanto, la probabilidad de que una partícula continúe es la
coeficiente de transmisión47

(2.142)

Por supuesto, el suma de estas probabilidades debe ser 1, y es:

(2.143)

Darse cuenta de R y T son funciones de β, y por lo tanto (Ecuaciones 2.133 y 2.138) de mi:

(2.144)

Cuanto mayor sea la energía, mayor será la probabilidad de transmisión (lo que tiene sentido).

Todo esto está muy ordenado, pero hay una cuestión de principio pegajosa que no podemos ignorar por completo: estas

funciones de onda de dispersión no son normalizables, por lo que en realidad no representan posibles estados de partículas.

Conocemos la resolución de este problema: forme combinaciones lineales normalizables de los estados estacionarios, tal como

hicimos con la partícula libre; las verdaderas partículas físicas están representadas por los paquetes de ondas resultantes.

Aunque es sencillo en principio, este es un negocio complicado en la práctica, y en este punto es mejor pasar el problema a una

computadora.48 Mientras tanto, dado que es imposible crear una función de onda de partícula libre normalizable sin involucrar

una distancia de energías, R y T debe interpretarse como el aproximado probabilidades de reflexión y transmisión de partículas

con energías en el vecindad de mi.

Por cierto, podría parecerle peculiar que pudiéramos analizar un problema esencialmente dependiente del tiempo (la

partícula entra, se dispersa un potencial y vuela hacia el infinito) usando estacionario estados. Después de todo, (en ecuaciones

2.134 y 2.135) es simplemente una función sinusoidal compleja, independiente del tiempo, que se extiende (con amplitud

constante) hasta el infinito en ambas direcciones. Y, sin embargo, al imponer condiciones de contorno adecuadas a esta función

pudimos determinar la probabilidad de que una partícula (representada por unlocalizado

89
paquete de ondas) rebotaría o atravesaría el potencial. El milagro matemático detrás de esto es, supongo, el hecho de que al

tomar combinaciones lineales de estados repartidos por todo el espacio, y con una dependencia temporal esencialmente trivial,

podemosconstruir funciones de onda que se concentran alrededor de un punto (en movimiento), con un comportamiento

bastante elaborado en el tiempo (ver Problema 2,42).

Siempre que tengamos las ecuaciones relevantes sobre la mesa, veamos brevemente el caso de una función deltabarrera (

Figura 2.15). Formalmente, todo lo que tenemos que hacer es cambiar el signo de α. Esto mata el estado ligado, por supuesto

(Problema2.2). Por otro lado, los coeficientes de reflexión y transmisión, que dependen solo de, no se modifican. Es extraño

decirlo, ¡es tan probable que la partícula atraviese la barrera como el pozo!Clásicamente, por supuesto, una partícula no puede

atravesar una barrera infinitamente alta, independientemente de su energía. De hecho, los problemas de dispersión clásicos

son bastante aburridos: si , luego y - la partícula ciertamente

lo hace sobre; si entonces y - sube la colina hasta que se queda sin vapor, y luego
regresa de la misma manera que vino. Cuántico Los problemas de dispersión son mucho más ricos: la partícula tiene

alguna probabilidad distinta de cero de atravesar el potencial incluso si. A este fenómeno lo llamamostunelización; es el

mecanismo que hace posible gran parte de la electrónica moderna, sin mencionar los espectaculares avances en

microscopía. Por el contrario, incluso si existe la posibilidad de que la partícula se recupere, aunque no recomendaría

conducir por un acantilado con la esperanza de que la mecánica cuántica lo salve (ver Problema2,35).

Figura 2.15: La barrera de la función delta.

∗ Problema 2.22 Evalúe las siguientes integrales:

(a) .
(B) .
(C) .

∗ Problema 2.23 Las funciones delta viven bajo signos integrales y dos expresiones
y que involucran funciones delta se dice que son iguales si

para cada función (ordinaria) .


(a) Muestra esa

(2.145)

dónde C es una constante real. (Asegúrese de comprobar el caso dondeC es negativo.)

(B) Sea el función de paso:

90
(2.146)

(En el raro caso en el que realmente importa, ser 1/2.)


definimos Mostrar eso.

∗∗ Problema 2.24 Verifique el principio de incertidumbre para la función de onda en la ecuación

2.132. Insinuación: Calcular la puede ser complicado, porque la derivada de tiene un paso

discontinuidad en . Es posible que desee utilizar el resultado en Problema2.23(B). Parcial

respuesta: .

Problema 2.25 Compruebe que el estado límite de la función delta también (Ecuación

2.132) es ortogonal a los estados de dispersión (ecuaciones 2.134 y 2.135).

∗ Problema 2.26 ¿Cuál es la transformada de Fourier del ? Usando Plancherel's


teorema? Demuestre que

(2.147)

Comentario: Esta fórmula da apoplejía a cualquier matemático respetable.


Aunque la integral es claramente infinita cuando, no converge (a cero o
cualquier otra cosa) cuando, ya que el integrando oscila para siempre. Hay
formas de arreglarlo (por ejemplo, puede integrar de a e interpretar la ecuación
2.147 para significar el promedio valor de la integral finita, como . La fuente de
El problema es que la función delta no cumple con el requisito (integración cuadrada) del

teorema de Plancherel (ver nota al pie de página). 42). A pesar de esto, la ecuación

2.147 puede ser extremadamente útil si se maneja con cuidado.

∗∗ Problema 2.27 Considera el doble potencial de función delta

donde α y a son constantes positivas.

(a) Dibuja este potencial.


(B) ¿Cuántos estados ligados posee? Encuentre las energías permitidas, por
y para, y bosquejar las funciones de onda.
(C) ¿Cuáles son las energías de estado ligado en los casos límite (i) y (ii)
(sosteniendo α fijo)? Explique por qué sus respuestas son razonables,

comparación con el pozo de función delta única.

91
∗∗ Problema 2.28 Encuentre el coeficiente de transmisión, para el potencial en Problema 2,27.

92
2.6 El pozo cuadrado finito
Como último ejemplo, considere el finito pozo cuadrado

(2.148)

dónde es una constante (positiva) (Figura 2.16). Al igual que la función delta bien, este potencial admite tanto enlazado

estados (con y estados de dispersión (con. Primero veremos los estados ligados.

Figura 2.16: El pozo cuadrado finito (Ecuación 2.148).

En la región el potencial es cero, por lo que la ecuación de Schrödinger dice

dónde

(2.149)

es real y positivo. La solución general es (como, por lo , pero el primer término estalla

que la solución físicamente admisible es

(2.150)

En la región , y la ecuación de Schrödinger dice

dónde

(2.151)

A pesar de que mi es negativo, para los estados ligados, debe ser mayor que , por el viejo teorema
(Problema 2.2); asi quel también es real y positivo. La solución general es49

(2.152)

93
dónde C y D son constantes arbitrarias. Finalmente, en la región la soluciónes nuevamente cero; El general
el potencial

es, pero el segundo término explota (como con , entonces nos quedamos

(2.153)

El siguiente paso es imponer condiciones de contorno: y continuo en y . Pero nosotros podemos

Ahorre un poco de tiempo al notar que este potencial es una función par, por lo que podemos asumir sin pérdida de

generalidad que las soluciones son pares o impares (Problema 2.1(C)). La ventaja de esto es que solo necesitamos

imponer las condiciones de contorno en un lado (digamos, en; el otro lado es automático, ya que. Yo resolveré las

soluciones pares; tú puedes hacer las impares en el Problema2,29. El coseno es par (y el seno es impar), así que estoy

buscando soluciones de la forma

(2.154)

La continuidad de ,a , dice

(2.155)

y la continuidad de dice

(2.156)

Ecuación de división 2.156 por ecuación 2.155, encontramos eso

(2.157)

Esta es una fórmula para las energías permitidas, ya que κ y l son ambas funciones de mi. Para resolvermi, primero adoptamos una

notación más agradable:

(2.158)

Según ecuaciones 2.149 y 2.151, lee , asi que y ecuación 2.157

(2.159)

Esta es una ecuación trascendental para z (y por lo tanto para como una función de (que es una medida del
"Tamaño" del pozo). Puede resolverse numéricamente, usando una computadora, o gráficamente, trazando y
en la misma cuadrícula y buscando puntos de intersección (ver Figura 2.17). Dos casos limitantes

son de especial interés:

1. Pozo ancho y profundo. Si es muy grande (empujando la curva hacia arriba en el gráfico, y

desliza el cruce por cero, , a la derecha) las intersecciones ocurren justo debajo , con
norte impar; sigue (ecuaciones2.158 y 2.151) ese

94
(2.160)

Pero es la energia sobre el fondo del pozo, y en el lado derecho tenemos precisamente el
energías de pozo cuadrado infinito, para un pozo de ancho (ver ecuación 2.30)-o mejor, mitad de ellos, desde
esta norte es impar. (Los otros, por supuesto, provienen delimpar funciones de onda, como descubrirá en el problema

2,29.) Así que el pozo del cuadrado finito pasa al pozo del cuadrado infinito, como; sin embargo, para cualquierfinito

sólo hay un número finito de estados ligados.

2. Pozo estrecho y poco profundo. A medida que disminuye, hay cada vez menos estados ligados, hasta que finalmente, para

, solo queda uno. Es interesante notar, sin embargo, que siempre hayuno estado obligado, no

importar cómo "débil ”el pozo se vuelve.

Figura 2.17: Solución gráfica de la ecuación 2.159, por (incluso estados).

Puedes normalizar, pasar (Ecuación 2.154), si está interesado (Problema 2.30), pero voy a
ahora a los estados de dispersión. . A la izquierda, donde tenemos

(2.161)

donde (como de costumbre)

(2.162)

Dentro del pozo, donde ,

(2.163)

donde, como antes,

(2.164)

A la derecha, asumiendo que no hay una onda entrante en esta región, tenemos

(2.165)

Aquí A es la amplitud incidente, B es la amplitud reflejada, y F es la amplitud transmitida.50


Hay cuatro condiciones de contorno: Continuidad de en dice

(2.166)

95
continuidad de a da

(2.167)

continuidad de a rendimientos

(2.168)

y continuidad de a requiere

(2.169)

Podemos usar dos de estos para eliminar C y Dy resuelve los dos restantes para B y F (ver problema 2,32):

(2.170)

(2.171)

El coeficiente de transmisión , expresada en términos de las variables originales, está dada por

(2.172)

Darse cuenta de (el pozo se vuelve "transparente") siempre que el seno es cero, es decir, cuando

(2.173)

dónde norte es cualquier número entero. Las energías para una transmisión perfecta, entonces, están dadas por

(2.174)

que resultan ser precisamente las energías permitidas para el infinito cuadrado bien. T se traza en la Figura 2.18, en función de

la energía.51

Figura 2.18: Coeficiente de transmisión en función de la energía (Ecuación 2.172).

∗ Problema 2.29 Analizar el impar funciones de onda de estado ligado para el pozo

cuadrado finito. Derive la ecuación trascendental para las energías permitidas y resuélvala

96
gráficamente. Examine los dos casos límite. ¿Existe siempre un estado límite extraño?

Problema 2.30 Normalizar en ecuación 2.154, para determinar las constantes D

y F.

Problema 2.31 La función delta de Dirac se puede considerar como el caso límite de un

rectángulo de área 1, ya que la altura llega al infinito y el ancho a cero. Demuestre que la

función delta también (Ecuación2.117) es un potencial "débil" (aunque es infinitamente

profundo), en el sentido de que. Determine la energía del estado ligado para el potencial

de función delta, tratándolo como el límite de un pozo cuadrado finito. Comprueba que tu

respuesta sea coherente con la ecuación2.132. También muestre que la ecuación2.172 se

reduce a la ecuación 2.144 en el límite apropiado.

Problema 2.32 Derivar ecuaciones 2.170 y 2.171. Insinuación: Usar ecuaciones 2.168 y

2.169 para resolver C y D en términos de F:

Vuelva a conectarlos en ecuaciones 2.166 y 2.167. Obtenga el coeficiente de


transmisión y confirme la ecuación2.172.

∗∗ Problema 2.33 Determine el coeficiente de transmisión para una rectangular.


barrera(igual que la ecuación 2.148, solo con en la región
. Trate por separado los tres casos ,y
(tenga en cuenta que la función de onda dentro de la barrera es diferente en los tres casos).

Respuesta parcial: por ,52

∗ Problema 2.34 Considere el potencial del "paso":53

(a) Calcule el coeficiente de reflexión, para el caso la y comentar sobre


respuesta.
(B) Calcule el coeficiente de reflexión para el caso. .

97
(C) Para un potencial (como éste) que no vuelve a cero a la derecha de la
barrera, el coeficiente de transmisión es no simplemente
(con A la amplitud del incidente y F la amplitud transmitida), porque la onda
transmitida viaja a diferentes velocidad. Muestra esa

(2.175)

por . Insinuación: Puedes resolverlo usando la ecuación 2,99, o más


elegantemente, pero de forma menos informativa, a partir de la corriente de probabilidad (Problema

2.18). Que esT, por ?


(D) Para, calcule el coeficiente de transmisión para el potencial de paso y
verifíquelo.

Problema 2.35 Una partícula de masa metro caída de potencial se acerca a un


abrupta de energía cinética (Figura 2.19).54

Figura 2.19: Dispersión desde un "acantilado" (Problema 2,35).

(a) ¿Cuál es la probabilidad de que se "refleje" de nuevo, si ? Insinuación:

Esto es como un problema 2,34, excepto que el paso ahora va abajo, en lugar de

hacia arriba.

(B) Dibujé la figura para hacerte pensar en un automóvil acercándose a un acantilado, pero

obviamente la probabilidad de "rebotar" desde el borde de un acantilado es lejosmás

pequeño que lo que obtuviste en (a), a menos que seas Bugs Bunny. Explique por qué

este potencialno representar correctamente un acantilado. Insinuación: En figura 2,20

la energía potencial del coche cae discontinuamente a, a medida que pasa

; ¿Sería esto cierto para un automóvil que se cae?

(C) Cuando un neutrón libre entra en un núcleo, experimenta una caída repentina de la energía

potencial, desde el exterior hasta alrededor de MeV (millones de electronvoltios) en el

interior. Supongamos que un neutrón, emitido con una energía cinética de 4 MeV por un

evento de fisión,

golpea tal núcleo. ¿Cuál es la probabilidad de que sea absorbido,


iniciando así otra fisión?Insinuación: Calculó la probabilidad de reflexión

98
en la parte (a); utilizar para obtener la probabilidad de transmisión a través de

la superficie.

Figura 2.20: El pozo cuadrado doble (Problema 2,47).

99
Más problemas en el capítulo 2

Problema 2.36 Resuelva la ecuación de Schrödinger independiente del tiempo con los

condiciones de contorno para el pozo cuadrado infinito "centrado": (para


(de lo contrario). Compruebe que sus energías permitidas

son consistentes con los míos (Ecuación 2.30), y confirme que su s puede
obtenerse de la mía (Ecuación 2,31) por la sustitución
(y renormalización adecuada). Dibuje sus primeras tres soluciones y compare la
Figura2.2. Tenga en cuenta que el ancho del pozo es ahora.

Problema 2.37 Una partícula en el pozo cuadrado infinito (Ecuación 2.22) tiene la inicial

función de onda

Determinar A, encontrar y calcular , en función del tiempo. Qué es


el valor esperado de la energía? Insinuación: y qué se puede reducir,
mediante la aplicación repetida de las fórmulas de suma trigonométrica,
a combinaciones lineales de y, con.

Problema 2.38

(a) Demuestre que la función de onda de una partícula en el cuadrado infinito vuelve a su

forma original después de un cuanto tiempo de avivamiento .


Es decir: para cualquier estado (no solo un estado estacionario).
(B) Cuál es el clásico tiempo de avivamiento, por una partícula de energía mi rebotando de un lado

a otro entre las paredes?

(C) ¿Para qué energía son iguales los dos tiempos de avivamiento?55

Problema 2.39 En problema 2,7(d) obtuvo el valor esperado de la energía por


sumando la serie en la ecuación 2.21, pero te advertí (en la nota al pie 21) no

intentarlo a la "manera antigua", , porque el


primera derivada discontinua de da la segunda derivada
problemático. De hecho, tupodría Lo he hecho usando la integración por partes, pero la

función delta de Dirac ofrece una forma mucho más limpia de manejar tales anomalías.

(a) Calcule la primera derivada de la respuesta en (en problema 2,7), y expresa el


términos de la función escalonada, , definido en la ecuación

2.146.
(B) Explotar el resultado del problema 2.23(b) escribir la segunda derivada de
en términos de la función delta.

(C) Evalúa la integral con la misma y compruebe que obtiene

respuesta que antes.

100

También podría gustarte